Sei sulla pagina 1di 40

29. JENNY AGABON, ET AL VS NLRC- Toni intended date of termination.

Respondents filed
complaints for illegal dismissal, underpayment of
Facts: Riviera Home Improvements, Inc. is engaged wages and nonpayment of service incentive leave
in the business of selling and installing ornamental and 13th month pay against JAKA. The Labor Arbiter
and construction materials. It employed Virgilio rendered a decision declaring the termination illegal
Agabon and Jenny Agabon as gypsum board and and ordering JAKA to reinstate respondents with full
cornice installers on January 2, 1992 until February backwages, and separation pay if reinstatement is
23, 1999 when they were dismissed for not possible. The Court of Appeals reversed said
abandonment of work. The Agabons filed a decision and ordered respondent JAKA to pay
complaint for illegal dismissal before the LA, who petitioners separation pay equivalent to one (1)
ruled in their favor. The NLRC reversed on appeal. month salary, the proportionate 13th month pay
The CA sustained the NLRCs decision. The Agabons and, in addition, full backwages from the time their
further appealed to the SC, disputing the finding of employment was terminated.
abandonment, and claiming that the company did
not comply with the twin requirements of notice and
hearing. Issue: What are the legal implications of a situation
where an employee is dismissed for cause but such
Issue: WON the Agabons were illegally dismissed? dismissal was effected without the employers
compliance with the notice requirement under the
Ruling: No. The dismissal should be upheld because Labor Code.
it was established that the Agabons abandoned their
jobs to work for another company. Riviera Home, Ruling: It was established that there was ground for
however, did not follow the notice requirements and respondents dismissal, i.e., retrenchment, which is
instead argued that sending notices to the last one of the authorized causes enumerated under
known addresses would have been useless because Article 283 of the Labor Code. Likewise, it is
they did not reside there anymore. Unfortunately, established that JAKA failed to comply with the
this is not a valid excuse because the law mandates notice requirement under the same Article.
the twin notice requirements to the employees last Considering the factual circumstances in the instant
known address. Thus, it should be held liable for case, the Court deem it proper to fix the indemnity
non-compliance with the procedural requirements at P50, 000.00. The Court of Appeals have been in
of due process. error when it ordered JAKA to pay respondents
separation pay equivalent to one (1) month salary
When the dismissal is for a just cause, the lack of for every year of service. In all cases of business
statutory due process should not nullify the closure or cessation of operation or undertaking of
dismissal, or render it illegal, or ineffectual. the employer, the affected employee is entitled to
However, the employer should indemnify the separation pay. This is consistent with the state
employee for the violation of his statutory rights. policy of treating labor as a primary social economic
force, affording full protection to its rights as well as
30. JAKA FOOD PROCESSING CORPORATION, vs.
its welfare. The exception is when the closure of
DARWIN PACOT, ROBERT PAROHINOG, DAVID
business or cessation of operations is due to serious
BISNAR, MARLON DOMINGO, RHOEL LESCANO and
business losses or financial reverses; duly proved, in
JONATHAN CAGABCAB. G.R. No. 151378. March 28,
which case, the right of affected employees to
2005- Julius
separation pay is lost for obvious reasons.
Facts: Respondents were earlier hired by petitioner
31. Abbott vs. Alcaraz- Juneey
JAKA Foods Processing Corporation until the latter
terminated their employment because the Facts: On June 27, 2004, Abbott Laboratories,
corporation was in dire financial straits. It is not Philippines (Abbott) caused the publication in a
disputed, however, that the termination was major broadsheet newspaper of its need for a
effected without JAKA complying with the Medical and Regulatory Affairs Manager. Alcaraz
requirement under Article 283 of the Labor Code showed interest and submitted her application on
regarding the service of a written notice upon the October 4, 2004. On December 7, 2004, Abbott
employees and the Department of Labor and formally offered Alcaraz the above-mentioned
Employment at least one (1) month before the
position which was an item under the companys performance evaluation discussed with her during
Hospira Affiliate Local Surveillance Unit (ALSU) the third and fifth months of her employment. Nor
department. In Abbotts offer sheet, it was stated did Abbott come up with the necessary Performance
that Alcaraz was to be employed on a probationary Improvement Plan to properly gauge Alcarazs
basis. Later that day, she accepted the said offer and performance with the set company standards. The
received an electronic mail (e-mail) from Abbotts Court modified Agabon v. NLRC in the case of Jaka
Recruitment Officer, Teresita C. Bernardo Food Processing Corporation v. Pacot where it
(Bernardo), confirming the same. Attached to created a distinction between procedurally defective
Bernardos e-mail were Abbotts organizational chart dismissals due to a just cause, on one hand, and
and a job description of Alcarazs work. On February those due to an authorized cause, on the other. If
12, 2005, Alcaraz signed an employment contract the dismissal is based on a just cause under Article
which stated that she was to be placed on probation 296 of the Labor Code but the employer failed to
for a period of six (6) months beginning February 15, comply with the notice requirement, the sanction to
2005 to August 14, 2005. On March 3, 2005, Maria be imposed upon him should be tempered because
Olivia T. Yabut-Misa, Abbotts Human Resources (HR) the dismissal process was, in effect, initiated by an
Director, sent Alcaraz an e-mail which contained an act imputable to the employee
explanation of the procedure for evaluating the
performance of probationary employees and further If the dismissal is based on an authorized cause
indicated that Abbott had only one evaluation under Article 297 but the employer failed to comply
system for all of its employees. Alcaraz was also with the notice requirement, the sanction should be
given copies of Abbotts Code of Conduct and stiffer because the dismissal process was initiated by
Probationary Performance Standards and Evaluation the employers exercise of his management
(PPSE) and Performance Excellence Orientation prerogative.
Modules (Performance Modules) which she had to
apply in line with her task of evaluating the Hospira Alcarazs dismissal proceeded from her failure to
ALSU staff. On May 23, 2005, Walsh, Almazar, and comply with the standards required for her
Bernardo personally handed to Alcaraz a letter regularization. As such, it is undeniable that the
stating that her services had been terminated dismissal process was, in effect, initiated by an act
effective May 19, 2005. Alcaraz felt that she was imputable to the employee, akin to dismissals due to
unjustly terminated from her employment and thus, just causes under Article 296 of the Labor Code.
filed a complaint for illegal dismissal and damages
against Abbott and its officers, namely, Misa,
Bernardo, Almazar, Walsh, Terrible, and Feist. She 32. ALU-TICP vs NLRC-Mamugay
claimed that she should have already been
considered as a regular and not a probationary Facts: National Steel Corporation (NSC) employed
employee given Abbotts failure to inform her of the petitioners in connection with its Five Year
reasonable standards for her regularization upon her Expansion Program. It undertook this program with
engagement as required under Article 295 of the the end in view of expanding the volume and
Labor Code. In this relation, she contended that increasing the kinds of products that it may offer for
while her employment contract stated that she was sale to the public.
to be engaged on a probationary status, the same
did not indicate the standards on which her The expansion program would consist of the
regularization would be based. ff.projects: 1. setting up ofa cold rolling mill
expansion project; 2. establishment of a billet steel-
Issue: WON Alcaraz was validly terminated from her making plant; 3. acquisition andinstallation of a Five-
employment stand TDM; and 4. the Cold Mill Peripherals Project.

Ruling: NO. Abbott failed to follow the above-stated Instead of contracting out with an outside or
procedure in evaluating Alcaraz. For one, there lies a independent contractor the tasks of constructing the
hiatus of evidence that a signed copy of Alcarazs buildings with related civil and electrical works that
PPSE form was submitted to the HRD. It was not would house the new machineries and equipment,
even shown that a PPSE form was completed to the company chose to execute and carry out the
formally assess her performance. Neither was the expansion projects in house, as it were, by
administration. The carrying out of this program whose services were terminated by reason of
constitutes a distinct undertaking identifiable from retrenchment or those who voluntarily resigned.
the ordinary business activity of the company (NSC is Castro filed a complaint for illegal dismissal against
not in the business of construction). Cosmos Bottling Corporation with the Labor Arbiter
contending that being a regular employee, he could
Thus, the components of the program, to which not be dismissed without a just and valid cause. The
complainants were assigned, were distinguishable company alleged that Castro was a mere project
from regular business of NSC which is production of employee whose employment was co-terminous
steel products. with the project for which he was hired.
Petitioners filed a complaint for unfair labor Issue: Whether or not private respondent Gil C.
practice, regularization and monetary benefits. Their Castro was illegally dismissed?
contention was that they should be considered
regular employees because their jobs are necessary, Ruling:
desirable and work related to NSCs main business
which is steel making and that they have rendered No. Castro being a project employee, or to use the
service for more than six years. correct term, seasonal employee, considering that
his employment was limited to the installation and
Issue: Are the petitioners entitled to such benefits? dismantling of petitioners annex plant machines
after which there was no more work to do, his
Ruling: No. Petitioners are project employees. As employment legally ended upon completion of the
discussed above, the nature of their work is not that project. That being the case, the termination of his
of a regular employee. Besides, each component employment cannot and should not constitute an
project, of course, begins and ends at specified times illegal dismissal. Neither should it constitute
which had already been determined by the time retrenchment as private respondent was a seasonal
petitioners were engaged. employee whose services were already terminated
on May 21, 1990 prior to the termination of the
Issue: Can the service of more than 6 years make other regular employees of Cosmos by reason of
them regular? retrenchment.
Ruling: No. The simple fact that the employment of
the petitioners as project employees had gone
beyond 1 year does not detract from or legally 34. Purefoods v. NLRC 283 SCRA 136 [1997]- Krizza
dissolve their status as project employees. The 1
year period in Article 280 of LC refers to casual Facts: The private respondents (numbering 906)
employees not project employees. were hired by petitioner Pure Foods Corporation to
work for a fixed period of five months at its tuna
33. Cosmos Bottling Corp., vs NLRC- June cannery plant. After the expiration of their
respective contracts of employment, their services
Facts: Gil C. Castro was employed by Cosmos were terminated. They then executed a Release
Bottling Corporation for a specific period. Having and Quitclaim stating that they had no claim
satisfactorily served the company for two (2) terms, whatsoever against Pure Foods. The private
Castro was recommended for reemployment with respondents filed before the NLRC Sub-Regional
the companys Maintenance Team for the Davao Arbitration Branch, a complaint for illegal dismissal
Project, he was re-hired and assigned to the against the petitioner and its plant manager. Pure
Maintenance Division of the Davao Project tasked to Foods Corp submits that the private respondents are
install the private respondents annex plant now estopped from questioning their separation
machines in its Davao plant. Castros employment from petitioners employ in view of their express
was terminated due to the completion of the special conformity with the five-month duration of their
project. Cosmos Bottling Corporation in valid employment contracts. Besides, they fell within the
exercise of its management prerogative terminated exception provided in Article 280 of the Labor Code
the services of some 228 regular employees by which reads: except where the employment has
reason of retrenchment. For obvious reasons, Castro been fixed for a specific project or undertaking the
was not among the list of those regular employees completion or termination of which has been
determined at the time of the engagement of the employed. In the instant case, the private
employee. Moreover, the first paragraph of the said respondents activities consisted in the receiving,
article must be read and interpreted in conjunction skinning, packing, and casing-up of tuna fish, which
with the proviso in the second paragraph, which were then exported by the petitioner. Indisputably,
reads: Provided that any employee who has they were performing activitieswhich were
rendered at least one year of service, whether such necessary and desirable in petitioners business or
service is continuous or broken, shall be considered trade.
a regular employee with respect to the activity in
which he is employed.... In the instant case, the ADDITIONAL DETAILS:
private respondents were employed for a period of
five months only. In any event, private respondents' Contrary to petitioner's submission, the private
prayer for reinstatement is well within the purview respondents could not be regarded as having been
of the Release and Quitclaim they had executed hired for a specific project or undertaking. The term
wherein they unconditionally released the petitioner specific project or undertaking under Article 280 of
from any and all other claims which might have the Labor Code contemplates an activity which is not
arisen from their past employment with the commonly or habitually performed or such type of
petitioner. work which is not done on a daily basis but only for a
specific duration of time or until completion; the
The private respondents, on the other hand, argue services employed are then necessary and desirable
that contracts with a specific period of employment in the employers usual business only for the period
may be given legal effect provided, however, that of time it takes to complete the project. The fact
they are not intended to circumvent the that the petitioner repeatedly and continuously
constitutional guarantee on security of tenure. They hired workers to do the same kind of work as that
submit that the practice of the petitioner in hiring performed by those whose contracts had expired
workers to work for a fixed duration of five months negates petitioners contention that those workers
only to replace them with other workers of the same were hired for a specific project or undertaking only.
employment duration was apparently to prevent the
regularization of these so-called casuals, which is a Where from the circumstances it is apparent that
clear circumvention of the law on security of tenure. the periods have been imposed to preclude
acquisition of tenurial security by the employee,
Issue: Whether employees hired for a definite period they should be struck down or disregarded as
and whose services are necessary and desirable in contrary to public policy and morals.
the usual business or trade of the employer are
regular employees. NOTE: criteria under which term employment
cannot be said to be in circumvention of the law on
Ruling: We find the petition devoid of merit. Article security of tenure:
280 of the Labor Code defines regular and casual
(See codal) 1) The fixed period of employment was knowingly
and voluntarily agreed upon by the parties without
An employment shall be deemed to be casual if it is any force, duress, or improper pressure being
not covered by the preceding paragraph; Provided, brought to bear upon the employee and absent any
That, any employee who has rendered at least one other circumstances vitiating his consent; or
year of service, whether such service is continuous
or broken, shall be considered a regular employee 2) It satisfactorily appears that the employer and the
with respect to the activity in which he is employed employee dealt with each other on more or less
and his employment shall continue while such equal terms with no moral dominance exercised by
activity exists. Thus, the two kinds of regular the former or the latter.
employees are (1) those who are engaged to
None of these criteria had been met in the present
perform activities which are necessary or desirable
case.This scheme of the petitioner was apparently
in the usual business or trade of the employer; and
designed to prevent the private respondents and the
(2) those casual employees who have rendered at
other casual employees from attaining the status
least one year of service, whether continuous or
of a regular employee. It was a clear circumvention
broken, with respect to the activity in which they are
of the employees right to security of tenure and to
other benefits like minimum wage, cost-of-living Issue:Whether or not the complainants were illegal
allowance, sick leave, holiday pay, and 13th month dismissed by PFVII
pay. Indeed, the petitioner succeeded in evading the
application of labor laws. Also, it saved itself from Ruling: By the very nature of things in a business
the trouble or burden of establishing a just cause for enterprise like PFVII, the services of the
terminating employees by the simple expedient of complainants are, indeed, more than six (6) months
refusing to renew the employment contracts.The a year. The company did not confine itself just to the
five-month period specified in private respondents processing of tomatoes and mangoes. It also
employment contracts having been imposed processed guyabano, calamansi, papaya, pineapple,
precisely to circumvent the constitutional guarantee etc. Besides, they have the office of administrative
on security of tenure should, therefore, be struck functions, cleaning and upkeeping of machines and
down or disregarded as contrary to public policy or other duties and tasks to keep up a big food
morals processing corporation. Considering, therefore, that
under Article 280 of the Labor Code "the provisions
35. Phil. Fruit and Vegetable Industries v. NLRC of written agreement to the contrary
[1999]-DURON notwithstanding and considering further that the
tasks which complainants performed were usually
Facts:Private respondent Philippine Fruit and necessary and desirable in the employers usual
Vegetable Workers Union-Tupas Local Chapter, for business or trade, Supreme Court held that
and in behalf of 127 of its members, filed a complainants are regular seasonal employees, thus,
complaint for unfair labor practice and/or illegal entitled to security of tenure. The Labor Code
dismissal with damages against Petitioner provides an employment shall be deemed to be
Corporation.They alleged that the dismissals were regular where the employee has been engaged to
due to complainants' involvement in union activities perform activities that are usually necessary or
and were without just cause. Labor Arbiter rendered desirable in the usual business or trade of the
judgment that Philippine Fruit & Vegetable employers, except where the employment has been
Industries, Inc (PFVII) were indeed guilty of Illegal fixed for a specific project. An employment shall be
Dismissal. On appeal, NLRC set aside the Labor deemed to be casual if it is not covered by the
Arbiters decision and remanded the said case to the preceding paragraph; provided, that, any employee
Arbitration Branch for further proceedings. who has rendered at least one year of service
Arbitration Branch rendered a decision finding PFVII whether such service is continuous or broken, shall
liable for illegal dismissal. On appeal, NLRC affirmed be considered a regular employee with respect to
the Arbitration Branchs decision but modified the the activity in which he is employed and his
awards of attorneys fees. PFVII filed a motion for employment shall continue while such actually
reconsideration which was denied by NLRC exists.

In the case at bar, the work of complainants as


One of PFVIIs contentions is that the
seeders, operators, sorters, slicers, janitors, drivers,
complainants are seasonal workers.
truck helpers, mechanics and office personnel is
According to them, its operation starts only
without doubt necessary in the usual business of a
in February with the processing of
food processing company like petitioner PFVII. It
tomatoes into tomato paste and ceases by
should be noted that complainants' employment has
the end of the same month when the
not been fixed for a specific project or undertaking
supply is consumed. It then resumes
the completion or termination of which has been
operations at the end of April or early May,
determined at the time of their appointment or
depending on the availability of supply with
hiring. Neither is their employment seasonal in
the processing of mangoes into purees and
nature. While it may be true that some phases of
ceases operation in June. The severance of
petitioner company's processing operations is
complainants' employment from petitioner
dependent on the supply of fruits for a particular
corporation was a necessary consequence
season, the other equally important aspects of its
of the nature of seasonal employment; and
business, such as manufacturing and marketing are
since complainants are seasonal workers as
not seasonal. The fact is that large-scale food
defined by the Labor Code, they cannot
processing companies such as Petitioner Company
invoke any tenurial benefit
continue to operate and do business throughout the declined to 2.8. Velayo recommended to the
year even if the availability of fruits and vegetables is petitioner that the respondents employment be
seasonal. PETITION DENIED. COMPLAINANTS ARE terminated due to habitual absenteeism, in
REGULAR EMPLOYEES BY VIRTUE OF THE FACT THAT accordance with the Company Rules and
THEY PERFORMED FUNCTIONS, WHICH ARE Regulations. Thus, the respondents contract of
NECESSARY AND DESIRABLE IN THE USUAL BUSINESS employment was no longer renewed.
OF PFVII.
ISSUE:

WON the respondent was still a contractual


36. PHILIPS SEMICONDUCTORS (PHILS.), INC., vs. employee of the petitioner as of June 4, 1993?
ELOISA FADRIQUELA- Karen
HELD:
Facts: Philips Semiconductors is a domestic
corporation engaged in the production and assembly No. Article 280 of the Labor Code of
of semiconductors such as power devices, RF the Philippines was emplaced in our statute books to
modules, CATV modules, RF and metal transistors prevent the circumvention by unscrupulous
and glass diods. Aside from contractual employees, employers of the employees right to be secure in his
the petitioner employed 1,029 regular workers. The tenure by indiscriminately and completely ruling out
employees were subjected to periodic performance all written and oral agreements inconsistent with the
appraisal based on output, quality, attendance and concept of regular employment defined therein.The
work attitude. One was required to obtain a language of the law manifests the intent to protect
performance rating of at least 3.0 for the period the tenurial interest of the worker who may be
covered by the performance appraisal to maintain denied the rights and benefits due a regular
good standing as an employee. employee because of lopsided agreements with the
economically powerful employer who can maneuver
Eloisa Fadriquela executed a Contract of to keep an employee on a casual or temporary
Employment with the petitioner in which she was status for as long as it is convenient to it. In tandem
hired as a production operator with a daily salary with Article 281 of the Labor Code, Article 280 was
of P118. Her initial contract was for a period of three designed to put an end to the pernicious practice of
months up to August 8, 1992, but was extended for making permanent casuals of our lowly employees
two months when she garnered a performance by the simple expedient of extending to them
rating of 3.15. Her contract was again renewed for temporary or probationary appointments, ad
two months or up to December 16, 1992, when she infinitum.
received a performance rating of 3.8. After the
expiration of her third contract, it was extended The two kinds of regular employees under the law
anew, for three months, that is, from January 4, are (1) those engaged to perform activities which are
1993 to April 4, 1993. necessary or desirable in the usual business or trade
of the employer; and (2) those casual employees
After garnering a performance rating of 3.4, the who have rendered at least one year of service,
respondents contract was extended for another whether continuous or broken, with respect to the
three months, that is, from April 5, 1993 to June 4, activities in which they are employed. The primary
1993. She, however, incurred five absences in the standard to determine a regular employment is the
month of April, three absences in the month of May reasonable connection between the particular
and four absences in the month of June. Line activity performed by the employee in relation to
supervisor Shirley F. Velayo asked the respondent the business or trade of the employer. The test is
why she incurred the said absences, but the latter whether the former is usually necessary or desirable
failed to explain her side. The respondent was in the usual business or trade of the employer. If the
warned that if she offered no valid justification for employee has been performing the job for at least
her absences, Velayo would have no other recourse one year, even if the performance is not continuous
but to recommend the non-renewal of her or merely intermittent, the law deems the repeated
contract. The respondent still failed to respond, as a and continuing need for its performance as sufficient
consequence of which her performance rating evidence of the necessity, if not indispensability of
that activity to the business of the employer. Hence, 37. Alcira vs NLRC, G. R. No. 149859, June 9, 2004-
the employment is also considered regular, but only Christal
with respect to such activity and while such activity
exists. The law does not provide the qualification Facts: Radin Alcira, was hired by the respondent
that the employee must first be issued a regular Middleby Philippines Corporation as engineering
appointment or must be declared as such before he support services supervisor under probationary
can acquire a regular employee status. status for 6 months. Afterwards, the service of the
petitioner was terminated by the respondent on the
In this case, the respondent was employed by the ground that the latter was not satisfied on the
petitioner on May 8, 1992 as production performance of the former. At certain occasions, he
operator. She was assigned to wirebuilding at the was late, absent and did not wear the prescribed
transistor division. There is no dispute that the work uniform. As a result, Alcira filed a complaint foe
of the respondent was necessary or desirable in the illegal dismissal in the National Labor Relations
business or trade of the petitioner. She remained Commission (NLRC) against the respondent.
under the employ of the petitioner without any Petitioner contended that his termination in the
interruption since May 8, 1992 to June 4, 1993 or for service tantamount to illegal dismissal since he
one (1) year and twenty-eight (28) days. The original attained the status of a regular employee as of the
contract of employment had been extended or time of dismissal. He presented the appointment
renewed for four times, to the same position, with paper showing that he was hired on May 20, 1996,
the same chores. Such a continuing need for the consequently, his dismissal on November 20, 1996
services of the respondent is sufficient evidence of was illegal because at that time, he was already a
the necessity and indispensability of her services to regular employee since the 6-month probationary
the petitioners business. By operation of law, then, period ended on November 16, 1996. The
the respondent had attained the regular status of respondent, on the other hand, asserted that during
her employment with the petitioner, and is thus the petitioners probationary period, he showed
entitled to security of tenure as provided for in poor performance on his assigned tasks, was late
Article 279 of the Labor Code which reads: couple of times and violated the companys rule.
Thus, the petitioner was terminated and his
Art. 279. Security of Tenure. In cases of regular application to become a regular employment was
employment, the employer shall not terminate the disapproved. The respondent also insisted that the
services of an employee except for a just cause or removal of the petitioner from office was within the
when authorized by this Title. An employee who is probationary period.
unjustly dismissed from work shall be entitled to
reinstatement without loss of seniority rights and Issue: Should the exact calendar days be counted to
other privileges and to his full backwages, inclusive determine the probationary period?
of allowances, and to his other benefits or their
monetary equivalent computed from the time his Held: No. Since, one month is composed of 30 days,
compensation was withheld from him up to the time then, 6 months shall be understood to be composed
of his actual reinstatement. of 180 days. And the computation of the 6- month
period is reckoned from the date of appointment up
The respondents re-employment under contracts to the same calendar date of the 6th month
ranging from two to three months over a period of following. Since, the number of days of a particular
one year and twenty-eight days, with an express month is irrelevant, petitioner was still a
statement that she may be reassigned at the probationary employee at the time of his dismissal.
discretion of the petitioner and that her Wherefore, the petition is dismissed. (Alcira counted
employment may be terminated at any time upon the exact calendar days in the month to reach 180
notice, was but a catch-all excuse to prevent her days, he did not count 30 days x 6, which is the right
regularization. Such statement is contrary to the method.)
letter and spirit of Articles 279 and 280 of the Labor
Code.
38. MITSUBISHI MOTORS PHILIPPINES CORP VS. employed on a probationary basis and was apprised
CHRYSLER PHILIPPINES LABOR UNION (R-U, Glenna) of the standards upon which his regularization would
be based during the orientation. His first day to
Facts: Private respondent Nelson Paras first worked report for work was on May 27, 1996. As per the
with Mitsubishi Philippines as a shuttle bus driver on company's policy, the probationary period was from
March 19, 1976. He resigned on June 16, 1982 three (3) months to a maximum of six (6) months.
because he went to Saudi Arabia and worked there Applying Article 13 of the Civil Code, the
as a diesel mechanic and heavy machine operator probationary period of six (6) months consists of one
from 1982 to 1993. Upon his return, Mitsubishi hundred eighty (180) days. The Court conforms with
Philippines re-hired him as a welder-fabricator at a paragraph one, Article 13 of the Civil Code providing
tooling shop from November 1, 1994 to March 3, that the months which are not designated by their
1995. On May 1996, Paras was re-hired again, this names shall be understood as consisting of thirty
time as a probationary manufacturing trainee at the (30) days each. This case, the Labor Code pertains to
Plant Engineering Maintenance Department. He had 180 days. Also, as clearly provided for in the last
an orientation on May 15, 1996 and afterwhich, with paragraph of Article 13, it is said that in computing a
respect to the companys rules and guidelines, period, the first day shall be excluded and the last
started reporting for work on May 27, 1996. Paras day included. Thus, the one hundred eighty (180)
was evaluated by his immediate supervisors after six days commenced on May 27, 1996, and ended on
months of working. The supervisors rating Paras November 23, 1996. The termination letter dated
performance were Lito R. Lacambacal and Wilfredo November 25, 1996 was served on respondent Paras
J. Lopez, as part of the MMPCs company policies. only at 3:00 a.m. of November 26, 1996. The Court
Upon this evaluation, Paras garnered an average held that by that time, he was actually already a
rating. Later, respondent Paras was informed by his regular employee of the petitioner under Article 281
supervisor, Lacambacal, that he received an average of the Labor Code. His position as a regularized
performance rating but it is a rate which would still employee is thus secured until further notice.
qualify him to be regularized. But as part of the
company protocols, the Division Managers namely
A.C. Velando, H.T. Victoria and Dante Ong reviewed
the performance evaluation made on Paras. Despite 39. PANGILINAN vs. GENERAL MILLING-Kate
the recommendations of the supervisors, they
unanimously agreed that the performance was FACTS: The respondent General Milling Corporation
unsatisfactory. As a consequence, Paras was not is a domestic corporation engaged in the production
considered for regularization. Paras received a and sale of livestock and poultry. It is, likewise, the
Notice of Termination on November 26, 1996 which distributor of dressed chicken to various restaurants
was dated November 25, 1996. This letters intent is and establishments nationwide. As such, it employs
to formally relieve him off of his services and hundreds of employees, some on a regular basis and
position effective the date since he failed to meet others on a casual basis, as emergency workers.
the companys standards. The petitioners were employed by the respondent
on different dates as emergency workers at its
Issue: Whether or not respondent Paras termination poultry plant in Cainta, Rizal, under separate
was legal or not. temporary/casual contracts of employment for a
period of five months. Most of them worked as
Held: The Court holds that a company employer may chicken dressers, while the others served as packers
indeed hire an employee on a probationary basis in or helpers. Upon the expiration of their respective
order to determine his fitness to perform work. The contracts, their services were terminated. They later
Court stresses the existence of the statements under filed separate complaints for illegal dismissal and
Article 281 of the Labor Code which specifies that non-payment of holiday pay, 13th month pay, night-
the employer must inform the employee of the shift differential and service incentive leave pay
standards they were to meet in order to be granted against the respondent before the Arbitration
regularization and that such probationary period Branch of the National Labor Relations Commission.
shall not exceed six (6) months from the date the
employee started working, unless specified in the
apprenticeship agreement. Respondent Paras was
The petitioners alleged that their work as chicken essentially contradictory between a definite period
dressers was necessary and desirable in the usual of employment and the nature of the employees
business of the respondent, and added that although duties.
they worked from 10:00 p.m. to 6:00 a.m., they were
not paid night-shift differential. They stressed that Stipulations in employment contracts providing for
based on the nature of their work, they were regular term employment or fixed period employment are
employees of the respondent; hence, could not be valid when the period were agreed upon knowingly
dismissed from their employment unless for just and voluntarily by the parties without force, duress
cause and after due notice. They asserted that the or improper pressure, being brought to bear upon
respondent GMC terminated their contract of the employee and absent any other circumstances
employment without just cause and due notice. vitiating his consent, or where it satisfactorily
They further argued that the respondent could not appears that the employer and employee dealt with
rely on the nomenclature of their employment as each other on more or less equal terms with no
temporary or casual. moral dominance whatever being exercised by the
former over the latter. An examination of the
ISSUE: Whether or not the petitioners were regular contracts entered into by the petitioners showed
employees of the respondent GMC when their that their employment was limited to a fixed period,
employment was terminated. usually five or six months, and did not go beyond
such period. The records reveal that the stipulations
HELD: The SC held the petitioners were employees in the employment contracts were knowingly and
with a fixed period, and, as such, were not regular voluntarily agreed to by the petitioners without
employees. Article 280 of the Labor Code force, duress or improper pressure, or any
comprehends three kinds of employees: (a) regular circumstances that vitiated their consent. Similarly,
employees or those whose work is necessary or nothing therein shows that these contracts were
desirable to the usual business of the employer; (b) used as a subterfuge by the respondent GMC to
project employees or those whose employment has evade the provisions of Articles 279 and 280 of the
been fixed for a specific project or undertaking the Labor Code.
completion or termination of which has been
determined at the time of the engagement of the The petitioners were hired as emergency workers
employee or where the work or services to be and assigned as chicken dressers, packers and
performed is seasonal in nature and the helpers at the Cainta Processing Plant. While the
employment is for the duration of the season; and, petitioners employment as chicken dressers is
(c) casual employees or those who are neither necessary and desirable in the usual business of the
regular nor project employees. respondent, they were employed on a mere
temporary basis, since their employment was limited
A regular employee is one who is engaged to to a fixed period. As such, they cannot be said to be
perform activities which are necessary and desirable regular employees, but are merely contractual
in the usual business or trade of the employer as employees. Consequently, there was no illegal
against those which are undertaken for a specific dismissal when the petitioners services were
project or are seasonal.[41] There are two separate terminated by reason of the expiration of their
instances whereby it can be determined that an contracts. Lack of notice of termination is of no
employment is regular: (1) if the particular activity consequence, because when the contract specifies
performed by the employee is necessary or desirable the period of its duration, it terminates on the
in the usual business or trade of the employer; and, expiration of such period. A contract for
(2) if the employee has been performing the job for employment for a definite period terminates by its
at least a year. Article 280 of the Labor Code does own term at the end of such period. Petition is
not proscribe or prohibit an employment contract denied.
with a fixed period. It does not necessarily follow
that where the duties of the employee consist of
activities usually necessary or desirable in the usual
business of the employer, the parties are forbidden
from agreeing on a period of time for the
performance of such activities. There is thus nothing
40. RAVAGO vs ESSO EASTERN MARINE, G.R. No. of the disability he suffered which rendered him
158324. March 14, 2005- Garry unfit to work as a seafarer. This fact was further
validated by the company doctor and Ravagos
FACTS: Esso Eastern Marine, now the Petroleum attending physician. They averred that Ravago was a
Shipping Ltd. is a foreign company based in contractual employee and was hired under 34
Singapore and engaged in maritime commerce. It is separate contracts by different companies. Ravago
represented in the Philippines by its manning agent insisted that he was fit to resume pre-injury
and co-respondent Trans-Global, a corporation activities and that he was not a mere contractual
organized under the Philippine laws. Roberto Ravago employee because the respondents regularly and
was hired by Trans-Global to work as a seaman on continuously rehired him for 23 years and, for his
board various Esso vessels. On February 13, 1970, continuous service, was awarded a CEIP payment
Ravago commenced his duty as S/N wiper on board upon his termination from employment.
the Esso Bataan under a contract that lasted until
February 10, 1971. Thereafter, he was assigned to ISSUE: Whether or not petitioner Ravago is a regular
work in different Esso vessels where he was employee of respondent Esso
designated diverse tasks, such as oiler, then assistant
engineer. He was employed under a total of 34 HELD: The SC held that seafarers are contractual, not
separate and unconnected contracts, each for a regular, employees. Seamen and overseas contract
fixed period, by three different companies, namely, workers are not covered by the term regular
Esso Tankers, Inc. (ETI), EEM and Esso International employment as defined in Article 280 of the Labor
Shipping (Bahamas) Co., Ltd. (EIS), Singapore Branch. Code. Clearly, petitioner cannot be considered as a
Ravago worked with Esso vessels until August 22, regular employee notwithstanding that the work he
1992, a period spanning more than 22 years. performs is necessary and desirable in the business
of respondent company. As expounded,
Shortly after completing his latest contract with an exception is made in the situation of
Esso, Ravago was granted a vacation leave with pay. seafarers. The exigencies of their work necessitates
Preparatory to his embarkation under a new that they be employed on a contractual basis.
contract, he was ordered to report for a Medical Pre-
Employment Examination, which, according to the 41.Hacienda Bino/Hortencia Starke vs. Cuenca ,G.R.
records, he passed. He, likewise, attended a Pre- No. 150478, April 15, 2005- SAL
Departure Orientation Seminar conducted by the
Capt. I.P. Estaniel Training Center, a division of Facts: Hacienda Bino, represented by owner and
Trans-Global. One night, a stray bullet hit Ravago on operator Hortencia L. Starke, is a 236- hectare sugar
the left leg while he was waiting for a bus ride in plantation in Kabankalan City. The 76 individual
Cubao, Quezon City. He fractured his left proximal respondents were part of its workforce, consisting of
tibia and was hospitalized at the Philippine 220 workers, performing various tasks in the
Orthopedic Hospital. Ravagos wife, Lolita, informed planting and harvesting of sugarcane. In 1996,
the petitioners of the incident for purposes of during the off-milling season, Starke issued an Order
availing medical benefits. As a result of his injury, of Notice to the employees, which stated: Please
Ravagos doctor opined that he would not be able to bear in mind that all those who signed in favor of
cope with the job of a seaman and suggested that he CARP are expressing their desire to get out of
be given a desk job. For this reason, the company employment on their own volition. Wherefore,
physician found him to have lost his dexterity, beginning today, July 18, only those who did not sign
making him unfit to work once again as a seaman. for CARP will be given employment by Hda. Bino.
Consequently, instead of rehiring Ravago, Esso paid Respondents filed a complaint for illegal dismissal,
him his Career Employment Incentive Plan (CEIP) as wage differentials, 13th month pay, holiday pay and
of and his final tax refund. However, Ravago filed a premium pay for holiday, service incentive leave pay,
complaint for illegal dismissal with prayer for and moral and exemplary damages with the NLRC. In
reinstatement, backwages, damages and attorneys their Joint Sworn Statement, they alleged that they
fees against Trans-Global and Esso with the POEA were dismissed because they applied as
Adjudication Office. Respondents denied that beneficiaries under the Comprehensive Agrarian
Ravago was dismissed without notice and just cause. Reform Program (CARP) over the land owned by
Rather, his services were no longer engaged in view Starke.
The Labor Arbiter found that Starkes notice was planting of rice and sugar cane thereon could not
tantamount to a termination of the respondents possibly entail a whole year operation. The herein
services, thus guilty of illegal dismissal. Petitioner case presents a different factual condition as the
was ordered to pay backwages and wage enormity of the size of the sugar hacienda of
differentials. The NLRC affirmed the decision of the petitioner, with an area of two hundred thirty-six
Labor Arbiter, with additional payment of holiday (236) hectares, simply do not allow for private
pay. The CA deleted the award for holiday pay and respondents to render work only for a definite
premium pay for holidays. period.

Issue: WON the CA violated the doctrine of stare The primary standard for determining regular
decisis laid down by the Supreme Court as to the employment is the reasonable connection between
status of the sugar workers. the particular activity performed by the employee in
relation to the usual trade or business of the
Ruling: NO. Starke contends that the CA violated the employer. There is no doubt that the respondents
doctrine of stare decisis in not applying the ruling in were performing work necessary and desirable in
Mercado vs. NLRC, which held that sugar workers the usual trade or business of an employer. Hence,
are seasonal employees, whose employment legally they can properly be classified as regular employees.
ends upon completion of the project or the season. For respondents to be excluded from those classified
Thus, the employment of respondents, being sugar as regular employees, it is not enough that they
workers, may be terminated at the end of the perform work or services that are seasonal in nature.
season and such termination cannot be considered They must have been employed only for the
an illegal dismissal. duration of one season. While the records
sufficiently show that the respondents work in the
Under the doctrine of stare decisis, when a court has hacienda was seasonal in nature, there was,
laid down a principle of law as applicable to a certain however, no proof that they were hired for the
state of facts, it will adhere to that principle and duration of one season only. In fact, the payrolls,
apply it to all future cases in which the facts are submitted in evidence by the petitioners, show that
substantially the same. Where the facts are they availed the services of the respondents since
essentially different, however, stare decisis does not 1991. Absent any proof to the contrary, the general
apply, for a perfectly sound principle as applied to rule of regular employment should, therefore, stand.
one set of facts might be entirely inappropriate
when a factual variance is introduced. The CA
correctly found that the facts involved in this case
are different from the Mercado case; therefore, the 42. G.R. No. 157214 June 7, 2005 PHILIPPINE
ruling in that case cannot be applied to the case at GLOBAL COMMUNICATIONS, INC., petitioner,vs.
bar. RICARDO DE VERA, respondent.(re: case#8)-CEL

Respondents in the instant case are REGULAR Facts:Respondent Ricardo De Vera is a physician by
EMPLOYEES. Although in the Mercado case, the profession whom petitioner enlisted to attend to the
Supreme Court held the sugar workers were not medical needs of its employees. Thru a letter De
regular but seasonal workers, nevertheless, the Vera, offered his services to the petitioner,
same does not operate to abandon the settled tproposing his plan of works required of a
doctrine that sugar workers are considered regular practitioner in industrial medicine. The parties
and permanent farm workers of a sugar plantation agreed and formalized respondent's proposal in a
owner. The disparity in facts between the Mercado document denominated as RETAINERSHIP
case and the instant case is best exemplified by the CONTRACT which will be for a period of one year
fact that the former decision ruled on the status of subject to renewal.Said contract was renewed
employment of farm laborers, who, as found by the yearly. The retainership arrangement went on from
labor arbiter, work only for a definite period for a 1981 to 1994 with changes in the retainer's fee.
farm worker, after which they offer their services to However, for the years 1995 and 1996, renewal of
other farm owners, considering the area in question the contract was only made verbally. However, in
being comparatively small, comprising of seventeen December 1996 when Philcom, thru a letter bearing
and a half (1712) hectares of land, such that the on the subject boldly written as "TERMINATION
RETAINERSHIP CONTRACT", informed De Vera of its hazardous in nature. As such, what applies here is
decision to discontinue the latter's "retainer's the last paragraph of Article 157 which, to stress,
contract with the Company effective at the close of provides that the employer may engage the services
business hours of December 31, 1996" because of a physician and dentist "on retained basis",
management has decided that it would be more subject to such regulations as the Secretary of Labor
practical to provide medical services to its may prescribe. The successive "retainership"
employees through accredited hospitals near the agreements of the parties definitely hue to the very
company premises. Both the NLRC and the CA ruled statutory provision relied upon by respondent.
that respondent is petitioners regular employee at
the time of his separation. Deeply embedded in our jurisprudence is the rule
that courts may not construe a statute that is free
Issue: Is respondent petitioners regular employee? from doubt. Where the law is clear and
unambiguous, it must be taken to mean exactly what
Held: No. The appellate courts premise that regular it says, and courts have no choice but to see to it
employees are those who perform activities which that the mandate is obeyed. As it is, Article 157 of
are desirable and necessary for the business of the the Labor Code clearly and unequivocally allows
employer is not determinative in this case. For, [the employers in non-hazardous establishments to
Court] take it that any agreement may provide that engage "on retained basis" the service of a dentist or
one party shall render services for and in behalf of physician. Nowhere does the law provide that the
another, no matter how necessary for the latters physician or dentist so engaged thereby becomes a
business, even without being hired as an employee. regular employee. The very phrase that they may be
This set-up is precisely true in the case of an engaged "on retained basis", revolts against the idea
independent contractorship as well as in an agency that this engagement gives rise to an employer-
agreement. Indeed, Article 280 of the Labor Code, employee relationship.
quoted by the appellate court, is not the yardstick
for determining the existence of an employment 43. INTEGRATED CONTRACTOR AND PLUMBING
relationship. As it is, the provision merely WORKS INC. VS NLRC- Toni
distinguishes between two (2) kinds of employees,
i.e., regular and casual. It does not apply where, as Facts: Integrated Contractor and plumbing works is a
here, the very existence of an employment plumbing contractor. Its business depends on the
relationship is in dispute. Buttressing his contention number and frequency of the projects it is able to
that he is a regular employee of petitioner, contract with its clients. Solon worked for petitioner,
respondent invokes Article 157 of the Labor Code, on February 23, 1998, while he was about to log out
and argues that he satisfies all the requirements. from work, he was informed by the warehouseman
that the main office had instructed them to tell him
Had only respondent read carefully the very it was his last day of work as he had been
statutory provision invoked by him, he would have terminated. When he went to the office on February
noticed that in non-hazardous workplaces, the 24, 1998 to verify his status, he found out that
employer may engage the services of a physician "on indeed, he had been terminated. He went back to
retained basis." As correctly observed by the Integrated Contractors office on February 27, 1998
petitioner, while it is true that the provision requires to sign a clearance so he could claim his 13th month
employers to engage the services of medical pay and tax refunds. However, he had second
practitioners in certain establishments depending on thoughts and refused to sign the clearance when he
the number of their employees, nothing is there in read the clearance indicating he had resigned. He
the law which says that medical practitioners so filed a complaint alleging that he was illegally
engaged be actually hired as employees, adding that dismissed without just cause and without due
the law, as written, only requires the employer "to process.
retain", not employ, a part-time physician who
needed to stay in the premises of the non-hazardous Issue: WON Solon is a project employee of the
workplace for 2 hours. petitioner or a regular employee and illegally
dismissed?
Respondent takes no issue on the fact that
petitioners business of telecommunications is not
Ruling: Solon is a regular employee. The test to the Labor Code and can only be removed for cause.
determine whether employment is regular or not is We found no valid cause attending to Solons
the reasonable connection between the particular dismissal and found also that his dismissal was
activity performed by the employee in relation to without due process.
the usual business or trade of the employer. Also, if
the employee has been performing the job for at Additionally, Article 277(b) of the Labor Code
least one year, even if the performance is not provides that
continuous or merely intermittent, the law deems
the repeated and continuing need for its ... Subject to the constitutional right of workers to
performance as sufficient evidence of the necessity, security of tenure and their right to be protected
if not indispensability of that activity to the business. against dismissal except for a just and authorized
Thus, we held that where the employment of project cause and without prejudice to the requirement of
employees is extended long after the supposed notice under Article 283 of this Code, the employer
project has been finished, the employees are shall furnish the worker whose employment is
removed from the scope of project employees and sought to be terminated a written notice containing
are considered regular employees. a statement of the causes for termination and shall
afford the latter ample opportunity to be heard and
While length of time may not be the to defend himself with the assistance of his
controlling test for project employment, it is vital in representative if he so desires in accordance with
determining if the employee was hired for a specific company rules and regulations promulgated
undertaking or tasked to perform functions vital, pursuant to guidelines set by the Department of
necessary and indispensable to the usual business or Labor and Employment
trade of the employer. Here, Solon had been a
project employee several times over. His The failure of the petitioner to comply with
employment ceased to be coterminous with specific these procedural guidelines renders its dismissal of
projects when he was repeatedly re-hired due to the private respondent, illegal. An illegally dismissed
demands of petitioners business. Where from the employee is entitled to reinstatement with full
circumstances it is apparent that periods have been backwages, inclusive of allowances, and to his other
imposed to preclude the acquisition of tenurial benefits computed from the time his compensation
security by the employee, they should be struck was withheld from him up to the time of his actual
down as contrary to public policy, morals, good reinstatement, pursuant to Article 279 of the Labor
customs or public order. Code.

Further, Policy Instructions No. 20 requires


employers to submit a report of an employees
44. Lacuesta v Ateneo De Manila G.R. No. 152777
termination to the nearest public employment office
(2005)-JAY
every time his employment was terminated due to a
completion of a project. The failure of the employer Issue: Whether petitioner was illegally dismissed?
to file termination reports is an indication that the
employee is not a project employee. Department Facts:Respondent hired, on a contractual basis,
Order No. 19 superseding Policy Instructions No. 20 petitioner as a part-time lecturer in its English
also expressly provides that the report of Department for the second semester of school year
termination is one of the indications of project 1988-1989. She was re-hired, still on a contractual
employment. In the case at bar, there was only one basis, for the first and second semesters of school
list of terminated workers submitted to the year 1989-1990. On July 13, 1990, the petitioner was
Department of Labor and Employment. If private first appointed as full-time instructor on probation,
respondent was a project employee, petitioner in the same department effective June 1, 1990 until
should have submitted a termination report for March 31, 1991. Thereafter, her contract as faculty
every completion of a project to which the former on probation was renewed effective April 1, 1991
was assigned. until March 31, 1992. She was again hired for a third
year effective April 1, 1992 until March 31, 1993.
As a regular worker, private respondent is During these three years she was on probation
entitled to security of tenure under Article 279 of status.
Later on, the petitioner was informed that her 45. POSEIDON FISHING vs NLRC- Julius
contract would no longer be renewed because she
did not integrate well with the English Department. FACTS: Private respondent was employed by
President of the Ateneo informed petitioner was not Poseidon Fishing in January 1988 as Chief Mate.
being terminated but her contract would simply After five years, he was promoted to Boat Captain. In
expire. Petitioner was offered as book editor but at 1999, petitioners, without reason, demoted
the same time applied for her clearance as respondent from Boat Captain to Radio Operator of
instructor. Upon expiry of her book editor contract, petitioner Poseidon. As a Radio Operator, he
she also processed her clearance. She then filed for monitored the daily activities in their office and
illegal dismissal. recorded in the duty logbook the names of the
callers and time of their calls.
Ruling: As previously held, a part-time teacher
cannot acquire permanent status. Only when one On 3 July 2000, private respondent failed to record a
has served as a full-time teacher can he acquire 7:25 a.m. call in one of the logbooks. However, he
permanent or regular status. The petitioner was a was able to record the same in the other logbook.
part-time lecturer before she was appointed as a Consequently, when he reviewed the two logbooks,
full-time instructor on probation. As a part-time he noticed that he was not able to record the said
lecturer, her employment as such had ended when call in one of the logbooks so he immediately
her contract expired. Thus, the three semesters she recorded the 7:25 a.m. call after the 7:30 a.m. entry.
served as part-time lecturer could not be credited to Around 9:00 oclock in the morning of 4 July 2000,
her in computing the number of years she has petitioner Jesus, the manager, detected the error in
served to qualify her for permanent status. the entry in the logbook. Subsequently, she asked
private respondent to prepare an incident report to
Completing the probation period does not explain the reason for the said oversight.
automatically qualify her to become a permanent
employee of the university. Petitioner could only At around 2:00 oclock in the afternoon of that same
qualify to become a permanent employee upon day, petitioner Poseidons secretary, summoned
fulfilling the reasonable standards for permanent private respondent to get his separation pay
employment as faculty member. Consistent with amounting to Fifty-Five Thousand Pesos
academic freedom and constitutional autonomy, an (P55,000.00). However, he refused to accept the
institution of higher learning has the prerogative to amount as he believed that he did nothing illegal to
provide standards for its teachers and determine warrant his immediate discharge from work.
whether these standards have been
met.http://sc.judiciary.gov.ph/jurisprudence/2005/d Private respondent then filed a complaint for illegal
ec2005/152777.htm - _ftn19 At the end of the dismissal with the Labor Arbiter. He averred that
probation period, the decision to re-hire an petitioner Poseidon employed him as a Chief Mate
employee on probation, belongs to the university as sometime in January 1988. He claimed that he was
the employer alone. promoted to the position of Boat Captain five years
after. However, in 1999, he was demoted from Boat
We reiterate, however, that probationary employees Captain to Radio Operator without any reason and
enjoy security of tenure, but only within the period shortly, he was terminated without just cause and
of probation. Likewise, an employee on probation without due process of law.
can only be dismissed for just cause or when he fails
to qualify as a regular employee in accordance with Conversely, petitioners Poseidon and Terry de Jesus
the reasonable standards made known by the strongly asserted that private respondent was a
employer at the time of his hiring. Upon expiration contractual or a casual employee whose services
of their contract of employment, academic could be terminated at the end of the contract even
personnel on probation cannot automatically claim without a just or authorized cause in view of Article
security of tenure and compel their employers to 280 of the Labor Code. Petitioners further posited
renew their employment contracts. In the instant that when the private respondent was engaged, it
case, petitioner, did not attain permanent status and was made clear to him that he was being employed
was not illegally dismissed. As found by the NLRC, only on a por viaje or per trip basis and that his
her contract merely expired. employment would be terminated at the end of the
trip for which he was being hired. As such, the tenurial protection of private respondent. Such
private respondent could not be entitled to pattern of re-hiring and the recurring need for his
separation pay and other monetary claims. services are testament to the necessity and
indispensability of such services to petitioners
ISSUE: Whether or not respondent Estoquia is a business or trade.
regular employee of petitioner.
Even if petitioners contention that its industry is
HELD: The SC held that the ruling in the Brent case seasonal in nature, once a project or work pool
could not apply in the case at bar. The acid test in employee has been: (1) continuously, as opposed to
considering fixed-term contracts as valid is: if from intermittently, re-hired by the same employer for
the circumstances it is apparent that periods have the same tasks or nature of tasks; and (2) these tasks
been imposed to preclude acquisition of tenurial are vital, necessary and indispensable to the usual
security by the employee, they should be business or trade of the employer, then the
disregarded for being contrary to public policy. The employee must be deemed a regular employee.
SC will not hesitate to nullify employment contracts
stipulating a fixed term after finding that the In fine, inasmuch as private respondents functions
purpose behind these contracts was to evade the as described above are no doubt usually necessary
application of the labor laws, since this is contrary to or desirable in the usual business or trade of
public policy. petitioner fishing company and he was hired
continuously for 12 years for the same nature of
Moreover, unlike in the Brent case where the period tasks, we are constrained to say that he belongs to
of the contract was fixed and clearly stated, note the ilk of regular employee. Being one, private
that in the case at bar, the terms of employment of respondents dismissal without valid cause was
private respondent as provided in the Kasunduan illegal.
was not only vague, it also failed to provide an actual
or specific date or period for the contract. There is 46. Abesco v. Ramirez- Juneey
nothing in the contract that says complainant, who
happened to be the captain of said vessel, is a FACTS: Petitioner company was engaged in a
casual, seasonal or a project worker. The date July 1 construction business where respondents were hired
to 31, 1998 under the heading Pagdating had been on different dates from 1976 to 1992 either as
placed there merely to indicate the possible date of laborers, road roller operators, painters or drivers. In
arrival of the vessel and is not an indication of the 1997, respondents filed two separate complaints for
status of employment of the crew of the vessel. illegal dismissal against the company and its General
Manager, Oscar Banzon, before the Labor Arbiter.
Furthermore, as petitioners themselves admitted in Petitioners allegedly dismissed them without a valid
their petition before this Court, private respondent reason and without due process of law. The
was repeatedly hired as part of the boats crew and complaints also included claims for non-payment of
he acted in various capacities onboard the vessel. the 13th month pay, five days service incentive
The test to determine whether employment is leave pay, premium pay for holidays and rest days,
regular or not is the reasonable connection between and moral and exemplary damages. The LA later on
the particular activity performed by the employee in ordered the consolidation of the two
relation to the usual business or trade of the complaints. Petitioners denied liability to
employer. And, if the employee has been performing respondents and countered that respondents were
the job for at least one year, even if the performance project employees since their services were
is not continuous or merely intermittent, the law necessary only when the company had projects to
deems the repeated and continuing need for its be completed. Petitioners argued that, being project
performance as sufficient evidence of the necessity, employees, respondents employment was
if not indispensability of that activity to the business. coterminous with the project to which they were
Ostensibly, in the case at bar, at different times, assigned. They were not regular employees who
private respondent occupied the position of Chief enjoyed security of tenure and entitlement to
Mate, Boat Captain, and Radio Operator. The act of separation pay upon termination from work.
hiring and re-hiring in various capacities is a mere
gambit employed by petitioner to thwart the
ISSUE: Whether respondents were project hired by the respondent to unload are basically
employees or regular employees. seasonal workers. They are hired whenever there
are trucks of suppliers do not have any
HELD: The SC held that respondents were regular accompanying truck boys. Whoever is available and
employees. The principal test for determining whoever are willing to help unload on a particular
whether employees are project employees or occasion are hired to unload. The complainants
regular employees is whether they are assigned to maintained that they are hired by Cebu Metal
carry out a specific project or undertaking, the Corparation as employees and filed a complaint with
duration and scope of which are specified at the the regional arbitration in Bacolod City for
time they are engaged for that project. Such underpayment of wages and non-payment of the
duration, as well as the particular work/service to be benefits and then claimed illegal dismissal because
performed, is defined in an employment agreement they were dismissed after the filing of the complaint.
and is made clear to the employees at the time of
hiring. The Labor Arbiter rendered a decision in favor of the
complainants. Aggrieved, Cebu Metal Corporation
In this case, petitioners did not have that kind of filed an appeal with the NLRC. The NLRC reversed
agreement with respondents. Neither did they and set aside the decision of the Labor Arbiter and
inform respondents of the nature of the latters held that the complainants were not regular
work at the time of hiring. Hence, for failure of employees, thus, they could not have been illegally
petitioners to substantiate their claim that dismissed.
respondents were project employees, we are
constrained to declare them as regular employees. The NLRC order of the reversal was based on the
Commissions finding that the petty cash vouchers
submitted by Cebu Metal Corporation confirmed the
fact that unloaders were paid on pakiao or task
47. Cebu Metal Corp. vs Saliling- Mamugay basis at Php 15.00 per metric ton. The Commission
further rationalized that with the irregular nature of
Facts: Cebu Metal Corporation is engaged in buying
the work involved in the stoppage and resumption of
and selling of scrap iron. In the Bacolod branch it has
which depended solely on the availability or supply
(3) regular employees holding such positions as
of scrap metal, it necessarily follows that after the
Officer-in-Charge, a scaler, and a yardman, whose
job of unloading was completed and unloaders are
salaries are paid directly by its main office in Cebu.
paid the contract price, the latters working
The complainants, Gregorio Saliling, Elias Bolido,
relationship with Cebu Metal Corporation legally
Manuel Alquiza, Benjie Amparado are the one who
ended. They were then free to offer their services to
undertakes pakiao work in the unloading of scrap
others.
iron. The Bacolod buying station is mainly a
stockyard where scrap metal delivered by its The complainants challenged the decision of the
suppliers are stockpiled. The supply of scrap metal is NLRC with the Court of Appeals, and it rendered the
not steady as it depends upon many factors, such as decision annulling the decision of the NLRC and
availability of supplies, price, competition and reinstated the decision made by the Labor Arbiter.
demand among others. There are weeks were there Hence, this petition.
are no delivery while there are weeks were a quite
of number of trucks are delivered to the stockyard. Issue: Were they illegally dismissed?
The arrivals of these trucks and the deliveries of
scrap metal iron are not regular and the schedules of Ruling: No. There can be no illegal dismissal to speak
deliveries to the stockyard are not known before of. Besides, the complainants cannot claim regularity
hand by the respondent Cebu Metal Corporation. in the hiring every time a truck comes loaded with
These trucks have their own driver and truck boys scrap metal. This is confirmed in the Petty Cash
employed by the different suppliers. Sometimes, Vouchers which are in the names of different leaders
these trucks do not have any truck boys, and in these who apportion the amount earned among its
instances, the corporation hires the services of members.
people for the unloading of the scrap metal from
these trucks. It is for this reason that the unloaders
And, quite telling is the fact that not every truck cause. Respondent could have easily proved that the
delivery of scrap metal requires the services of project or phase for which petitioner was hired has
respondent complainants when particular truck is already been completed. A certificate from the
accompanied by its own unloader. And whenever owner of the vessel serviced by the company,
required, respondent complainants were not always pictures perhaps, of the work accomplished, and
the ones contracted to undertake the unloading of other proof of completion could have been procured
the trucks since the work was offered to whomever by respondent. However, all that we have is
were available at a given time. respondent's self-serving assertion that the project
has been completed.
It should be remembered that the Philippine
Constitution, while inexorably committed towards The Court has held that an employment ceases to be
the protection of the working class from exploitation co-terminous with specific projects when the
and unfair treatment, nevertheless mandates the employee is continuously rehired due to the
policy of social justice so as to strike a balance demands of employer's business and re-engaged for
between an avowed predilection for labor, on the many more projects without interruption. In
one hand, and the maintenance of the legal rights of Maraguinot, Jr. v. NLRC (Second Division), the Court
capital, the proverbial hen that lays the golden egg, ruled that "once a project or work pool employee
on the other. has been: (1) continuously, as opposed to
intermittently, rehired by the same employer for the
same tasks or nature of tasks; and (2) these tasks are
vital, necessary and indispensable to the usual
48. Liganza v. RBL shipyard- June business or trade of the employer, then the
employee must be deemed a regular employee,
Facts: After working as a carpenter for respondent
pursuant to Article 280 of the Labor Code and
RBL since August 1991, petitioner Liganzas
jurisprudence."
employment was terminated on 30 October 1999.
Such event prompted petitioner to file a complaint Petitioner, as carpenter, was tasked to "make and
for illegal dismissal, alleging that on said date he was repair cabinet, flooring, quarters, ceiling, windows,
verbally informed that he was already terminated doors, kitchen and other parts of the vessel that
from employment and barred from entering the needs to be repaired." As such, petitioner's work
premises. On the same occasion, he was told to look was necessary or desirable to respondent's business.
for another job. Thus, he claimed that he was Assuming, without granting that petitioner was
unceremoniously terminated from employment initially hired for specific projects or undertakings,
without any valid or authorized cause. On the other the repeated re-hiring and continuing need for his
hand, respondent insisted that petitioner was a services for over eight (8) years have undeniably
mere project employee who was terminated upon made him a regular employee.
completion of the project for which he was hired.
Respondent capitalizes on the ruling in D.M.
Issue: WON Liganza was illegally dismissed? Consunji, Inc. v. NLRC which reiterates the rule that
the length of service of a project employee is not the
Ruling:
controlling test of employment tenure but whether
Yes. Even assuming that petitioner is a project or not "the employment has been fixed for a specific
employee, respondent failed to prove that his project or undertaking the completion or
termination was for a just and valid cause. While it is termination of which has been determined at the
true that the employment contract states that the time of the engagement of the employee."
contract ends upon a specific date, or upon
Surely, length of time is not the controlling test for
completion of the project, respondent failed to
project employment. Nevertheless, it is vital in
prove that the last project was indeed completed so
determining if the employee was hired for a specific
as to justify petitioner's termination from
undertaking or tasked to perform functions vital,
employment.
necessary and indispensable to the usual business or
In termination cases, the burden of proof rests on trade of the employer.
the employer to show that the dismissal is for a just
In the case at bar, respondent had been a project Accounts Specialists. The petitioners alleged that
employee several times over. His employment they were illegally dismissed by SMC.
ceased to be coterminous with specific projects
when he was repeatedly re-hired due to the The Labor Arbiter held petitioners were illegally
demands of petitioner's business. Where from the dismissed. The Decision of the Labor Arbiter was
circumstances it is apparent that periods have been affirmed on appeal by the NLRC. Court of Appeals
imposed to preclude the acquisition of tenurial reversed the decision of the Labor Arbiter and of the
security by the employee, they should be struck NLRC
down as contrary to public policy, morals, good
customs or public order. Issue: WON the termination of the petitioners is
valid?
The circumstances of this case reveal that there are
serious doubts in the evidence on record that Ruling: Article 280 of the Labor Code defines regular
petitioner is a project employee, or that he was employment(SEE DEFINITION
terminated for just cause. These doubts shall be
In Pure Foods Corp. v. NLRC, Supreme Court held
resolved in favor of petitioner, in line with the policy
that under the above-quoted provision, there are
of the law to afford protection to labor and construe
two kinds of regular employees, namely: (1) those
doubts in favor of labor.
who are engaged to perform activities which are
necessary or desirable in the usual business or trade
of the employer, and (2) those casual employees
49. Fabeza v. San Miguel Corp., G.R. No. 150658, who have rendered at least one year of service,
February 9, 2007- Krizza whether continuous or broken, with respect to the
activity in which they are employed.
Facts: Petitioners were hired by respondent San
Miguel Corporation (SMC) as "Relief Salesmen" for In Brent School, Inc. v. Zamora, the Supreme Court
the Greater Manila Area (GMA) under separate but laid out that Article 280 of the Labor Code appears
almost similarly worded "Contracts of Employment to prevent circumvention of the employees right to
With Fixed Period." After having entered into be secure in his tenure, the clause in said
successive contracts of the same nature with SMC, article indiscriminately and completely ruling out all
the services of petitioners, as well as de Lara and written or oral agreements conflicting with the
Alovera, were terminated after SMC no longer concept of regular employment as defined therein
agreed to forge another contract with them. SMC should be construed to refer to the substantive evil
and its co-respondent Arman Hicarte, who was its that the Code itself has singled out: agreements
Human Resources Manager, claimed that the hiring entered into precisely to circumvent security of
of petitioners was not intended to be permanent, as tenure. It should have no application to instances
the same was merely occasioned by the need to fill where a fixed period of employment was agreed
in a vacuum arising from SMCs gradual transition to upon knowingly and voluntarily by the parties,
a new system of selling and delivering its products. without any force, duress or improper pressure
While some of the qualified regular salesmen were being brought to bear upon the employee and
readily upgraded to the position of Accounts absent any other circumstances vitiating his consent,
Specialist, respondents claimed that SMC still had to or where it satisfactorily appears that the employer
sell its beer products using the conventional routing and employee dealt with each other on more or less
system during the transition stage, thus giving rise to equal terms with no moral dominance whatever
the need for temporary employees; and the being exercised by the former over the latter.
members of the regular Route Crew then existing
were required to undergo a training program to Since respondents attribute the termination of
determine whether they possessed or could be petitioners employment to the expiration of their
trained for the necessary attitude and aptitude respective contracts, a determination of whether
required of an Accounts Specialist, hence, the hiring petitioners were hired as project or seasonal
of petitioners and others for a fixed period, co- employees, or as fixed-term employees without any
terminus with the completion of the transition force, duress or improper pressure having been
period and Training Program for all prospective exerted against them is in order. If petitioners fall
under any of these categories, then indeed their
termination follows from the expiration of their Ruling: The Labor Arbiter, the NLRC, and the Court of
contracts. The NLRC had sufficient basis to believe Appeals all found that substantial evidence supports
that the shift of SMC to the Pre-Selling System was the absence of illegal dismissal in the present case.
not the real basis for the forging of fixed-term Article 283 of the Labor Code provides that an
contracts of employment with petitioners and that employer may dismiss from work an employee by
the periods were fixed only as a means to preclude reason of redundancy. The same provision also
petitioners from acquiring security of tenure. That states the procedural requirements for the validity
petitioners themselves insisted on the one-year of the dismissal. It is clear that the foregoing
fixed-term is not even alleged by respondents. In documentary evidence constituted substantial
fact, the sustained desire of each of the petitioners evidence to support the findings of Labor Arbiter
to enter into another employment contract upon the Lustria and the NLRC that petitioners employment
termination of the earlier ones clearly indicates their was terminated by respondent PLDT due to a valid or
interest in continuing to work for SMC. Moreover, legal redundancy program since substantial evidence
respondents have not established that the merely refers to that amount of evidence which a
engagement of petitioners services, which is not in reasonable mind might accept as adequate to
the nature of a project employment, required a support a conclusion. The records show that
definite date of termination as a sine qua non. respondent PLDT had sufficiently established the
existence of redundancy in the position of
Switchman. It is evident from the foregoing facts
that respondent PLDTs utilization of high technology
50. Soriano vs. NLRC, G.R. No. 165594, April 23, equipment in its operation such as computers and
2007, citing 2005 Filipina Pre-fabricated Bldg. digital switches necessarily resulted in the reduction
System (Filisystem)- Duron of the demand for the services of a Switchman since
computers and digital switches can aptly perform
Facts:Petitioner and certain individuals namely
the function of several Switchmen. Indubitably, the
Sergio Benjamin (Benjamin), Maximino Gonzales
position of Switchman has become redundant. As to
(Gonzales), and Noel Apostol (Apostol) were
whether Lazam was competent to testify on the
employed by the respondent as Switchman Helpers
effects of respondent PLDTs adoption of new
in its Tondo Exchange Office (TEO). After
technology vis--vis the petitioners position of
participating in several trainings and seminars,
Switchman, the records show that Lazam was highly
petitioner, Benjamin, and Gonzales were promoted
qualified to do so. He is a licensed electrical engineer
as Switchmen. Apostol, on the other hand, was
and has been employed by the respondent PLDT
elevated to the position of Frameman. One of their
since 1971. He was a Senior Manager for Switching
duties as Switchmen and Frameman was the manual
Division in several offices of the respondent PLDT,
operation and maintenance of the Electronic
and had attended multiple training programs on
Mechanical Device (EMD) of the TEO. In November
Electronic Switching Systems in progressive
1995, respondent PLDT implemented a company-
countries. He was also a training instructor of
wide redundancy program. Subsequently, the
Switchmen in the respondents office. The fact that
respondent PLDT gave separate letters dated 15 July
respondent PLDT hired contractual employees after
1996 to petitioner, Benjamin, Gonzales, and Apostol
implementing its redundancy program does not
informing them that their respective positions were
necessarily negate the existence of redundancy. As
deemed redundant due to the above-cited reasons
amply stated by the respondent PLDT, such hiring
and that their services will be terminated on 16
was intended solely for winding up operations using
August 1996.10 They requested the respondent
the old system. Since the respondent PLDT
PLDT for transfer to some vacant positions but their
determined that petitioners services are no longer
requests were denied since all positions were
necessary either as a Switchman or in any other
already filled up. Hence, on 16 August 1996,
position, and such determination was made in good
respondent PLDT dismissed the four from
faith and in furtherance of its business interest, the
employment.
petitioners contention that he should be the last
Issue: WON illegal dismissal is present in this case switchman to be laid-off by reason of his
qualifications and outstanding work must fail.
51 CASERES vs UNIVERSAL ROBINA SUGAR MILLING has rendered service for at least one (1) year shall be
CORP. (URSUMCO) -Karen considered a regular employee, pertains to
casual employees and not to project employees.
Facts: URSUMCO is a corporation engaged in the
cane sugar milling business. Pedy Caseres started
working for respondent in 1989, while Andito Pael in
1993. At the start of their respective employments, 52. PIER 8 ARRASTRE & STEVEDORING SERVICES,
they were made to sign a Contract of Employment INC. and/ or ELIODORO C. CRUZ- versus -JEFF B.
for Specific Project or Undertaking. Petitioners' BOCLOT, G.R. No. 173849, September 28, 2007-
contracts were renewed from time to time, until CHRISTAL
May 1999 when they were informed that their
contracts will not be renewed anymore. Facts: Boclot worked on and off as a stevedore at
PASSI. He only worked when there were other
Petitioners contend that respondent's repeated employees who could not report for work. He
hiring of their services qualifies them to the status of worked for an uncontinuos period in PASSi which
regular employees. totaled 228.5 days. He argued that he was a casual
worker since he started working at PASSI. He argued
ISSUE: WHETHER OR NOT THE PETITIONERS ARE thathe was supposed to be a regular employee
REGULAR EMPLOYEES OF RESPONDENTS? according to the CBA which states: The Company
agrees to convert to regular status all incumbent
HELD: No. The fact that petitioners were constantly probationary or casual employees and workers in
re-hired does not ipso facto establish that they the Company who have served the Company for an
became regular employees. Their respective accumulated service term of employment of not less
contracts with respondent show that there were than six (6) months from his original date of hiring.
intervals in their employment. In
petitioner Caseres's case, while his employment Issue: Whether or not he has attained regular
lasted from August 1989 to May 1999, the duration status?
of his employment ranged from one day to several
months at a time, and such successive employments Ruling: Under the CBA, he qualifies as a regular
were not continuous. With regard to petitioner Pael, employee The Supreme Court still finds respondent
his employment never lasted for more than a month to be a regular employee on the basis of pertinent
at a time. These support the conclusion that they provisions under the CBA between PASSI and its
were indeed project employees, and since their work Workers union, wherein it was stated that it agrees
depended on the availability of such contracts or to convert to regular status all incumbent
projects, necessarily the employment of probationary or casual employees and workers in
respondents work force was not permanent but co- the Company who have served the Company for an
terminous with the projects to which they were accumulated service term of employment of not less
assigned and from whose payrolls they were paid. As than six (6) months from his original date of hiring.
ruled in Palomares v. National Labor Relations Respondent assents that he is not a member of the
Commission,[18] it would be extremely burdensome union, as he was not recognized by PASSI as its
for their employer to retain them as regular employee, but this Court notes that PASSI
permanent employees and pay them wages even if adopts a union-shop agreement, culling from Article
there were no projects to work on. II of its CBA. Under a union-shop agreement,
although nonmembers may be hired, an employee is
Moreover, even if petitioners were repeatedly and required to become a union member after a certain
successively re-hired, still it did not qualify them as period, in order to retain employment. This
regular employees, as length of service is not the requirement applies to present and future
controlling determinant of the employment tenure employees. The same article of the CBA stipulates
of a project employee, but whether the employment that employment in PASSI cannot be obtained
has been fixed for a specific project or undertaking, without prior membership in the union. Hence,
its completion has been determined at the time of applying the foregoing provisions of the CBA,
the engagement of the employee. Further, the respondent should be considered a regular
proviso in Article 280, stating that an employee who employee after six months of accumulated service.
Having rendered 228.5 days, or eight months of whether the work undertaken is necessary or
service to petitioners since 1999, then respondent is desirable in the usual business or trade of the
entitled to regularization by virtue of the said CBA employer, a fact that can be assessed by looking into
provisions. the nature of the services rendered and its relation
to the general scheme under which the business or
Note: if there was no such stipulation in the CBA trade is pursued in the usual course. It is
agreement, he would not have been considered a distinguished from a specific undertaking that is
regular employee under the Labor Code. Although divorced from the normal activities required in
his work was necessary and desirable, he was a mere carrying on the particular business or trade.
reliever. He cannot also be a casual employee turned
regular since he did not work for at least a year ( he But, although the work to be performed is only for a
only worked for 228.5 days.) specific project or seasonal, where a person thus
engaged has been performing the job for at least
53. Pacquing et. al. vs. Coca-Cola Phils. (R-U, one year, even if the performance is not continuous
Glenna) or is merely intermittent, the law deems the
repeated and continuing need for its performance as
Facts:Petitioners were part of a complement of being sufficient to indicate the necessity or
three personnel comprised of a driver, a salesman desirability of that activity to the business or trade of
and a regular route helper, for every delivery truck. the employer. The employment of such person is
They worked exclusively at respondent's plants, sales also then deemed to be regular with respect to such
offices, and company premises. On October 22, activity and while such activity exists.
1996, petitioners filed a Complaint against
respondent for unfair labor practice and illegal The argument of petitioner that its usual business or
dismissal with claims for regularization, recovery of trade is softdrink manufacturing and that the work
benefits under the Collective Bargaining Agreement assigned to respondent workers as sales route
(CBA), moral and exemplary damages, and attorney's helpers so involves merely post production
fees. In their Position Paper, petitioners alleged that activities, one which is not indispensable in the
they should be declared regular employees of manufacture of its products, scarcely can be
respondent since the nature of their work as persuasive. If, as so argued by petitioner company,
cargadores-pahinantes was necessary or desirable to only those whose work are directly involved in the
respondent's usual business and was directly related production of softdrinks may be held performing
to respondent's business and trade. In its Position functions necessary and desirable in its usual
Paper, respondent denied liability to petitioners and business or trade, there would have then been no
countered that petitioners were temporary workers need for it to even maintain regular truck sales route
who were engaged for a five-month period to act as helpers. The nature of the work performed must be
substitutes for an absent regular employee. viewed from a perspective of the business or trade
in its entirety and not on a confined scope.
Issue: Whether petitioners are regular employees.
The repeated rehiring of respondent workers and
Applying the principle stare decisis et non quieta the continuing need for their services clearly attest
movere (follow past precedents and do not disturb to the necessity or desirability of their services in the
what has been settled), SC applied the ruling in regular conduct of the business or trade of
Magsalin v. National Organization of Working Men petitioner company.
to this case.
Being regular employees of respondent, petitioners
Held: The basic law on the case is Article 280, now are entitled to security of tenure, as provided in
295 (LCP). Article 279 of the Labor Code, and may only be
terminated from employment due to just or
In determining whether an employment should be
authorized causes. Because respondent failed to
considered regular or non-regular, the applicable
show such cause, the petitioners are deemed
test is the reasonable connection between the
illegally dismissed and therefore entitled to back
particular activity performed by the employee in
wages and reinstatement without loss of seniority
relation to the usual business or trade of the
rights and other privileges.
employer. The standard, supplied by the law itself, is
project employment contracts covering Viscas
alleged periods of employment. More importantly,
54. Cocomangas Hotel Beach Resort and/or Susan there is no evidence that Cocomangas reported the
Munro v. Federico Visca, Johnny Baredo, termination of Viscas supposed project employment
Ronald Tibus, Richard Visca, and Raffie Visca- KATE to the DOLE as project employees. Department
Order No. 19, as well as the old Policy Instructions
Facts: Visca et. al., who worked for the maintenance No. 20, requires employers to submit a report of an
and repairs of the petitioner, filed complaint against employees termination to the nearest public
Cocomangas for illegal dismissal. They alleged that employment office every time his employment is
they were regular employees, but were eventually terminated due to a completion of a project.
informed by Cocomangas not to report for work Cocomangas failure to file termination reports is an
because they caused irritation and annoyance to indication that Visca et. al. were not project
resorts guests, and budgetary constraints. employees but regular employees.
Eventually, they found out that Cocomangas hired An employment ceases to be coterminous with
new employees as their replacement. specific projects when the employee is continuously
Cocomangas alleged that there was no employer- rehired due to the demands of employers business
employee relationship, and that Visca was an and re-engaged for many more projects without
independent contractor who was called upon from interruption. Once a project or work pool employee
time to time when repairs were needed. has been: (1) continuously, as opposed to
Labor Arbiter ruled that Visca was an independent intermittently, rehired by the same employer for the
contractor, and that the other respondents were same tasks or nature of tasks; and (2) these tasks are
hired by him; and that there was no illegal dismissal vital, necessary and indispensable to the usual
but rather completion of projects; and that business or trade of the employer, then the
respondents were project workers, not regular employee must be deemed a regular employee,
employees. pursuant to Article 280 of the Labor Code and
NLRC affirmed the decision. CA reversed. jurisprudence.
Issue: 55. Price et. al. vs. Innodata, G.R. No. 178505- Garry
Whether or not Visca et. al. were regular employees.
FACTS: Respondent Innodata Philippines,
Held: Yes, Visca et. al. were regular employees. Inc./Innodata Corporation (INNODATA) was a
A project employee is one whose employment has domestic corporation engaged in the data encoding
been fixed for a specific project or undertaking, the and data conversion business. It employed encoders,
completion or termination of which has been indexers, formatters, programmers, quality/quantity
determined at the time of the engagement of the staff, and others, to maintain its business and
employee or where the work or service to be accomplish the job orders of its clients. INNODATA
performed is seasonal in nature and the had since ceased operations due to business losses
employment is for the duration of the season. in June 2002. Petitioners herein were employed as
Before an employee hired on a per-project basis can formatters by INNODATA. The parties executed an
be dismissed, a report must be made to the nearest employment contract denominated as a "Contract of
employment office, of the termination of the Employment for a Fixed Period," stipulating that the
services of the workers every time completes a contract shall be for a period of one year. During
project, pursuant to Policy Instruction No. 20. their employment as formatters, petitioners were
Visca cannot be classified as project employees, assigned to handle jobs for various clients. Once they
since they worked continuously for Cocomangas finished the job for one client, they were
from three to twelve years without any mention of a immediately assigned to do a new job for another
project to which they were specifically assigned. client. On 16 February 2000, the HRAD Manager of
While they had designations as foreman, carpenter INNODATA wrote petitioners informing them of their
and mason, they performed work other than last day of work. INNODATA averred that petitioners
carpentry or masonry. They were tasked with the employment already ceased due to the end of their
maintenance and repair of the furniture, motor contract.
boats, cottages, and windbreakers and other resort
facilities. There was likewise no evidence of the
Petitioners filed a Complaint for illegal dismissal and
damages against respondents. Petitioners claimed
that they should be considered regular employees After considering petitioners contracts in their
since their positions as formatters were necessary entirety, as well as the circumstances surrounding
and desirable to the usual business of INNODATA as petitioners employment at INNODATA, the Court is
an encoding, conversion and data processing convinced that the terms fixed therein were meant
company. Petitioners finally argued that they could only to circumvent petitioners right to security of
not be considered project employees considering tenure and are, therefore, invalid.
that their employment was not coterminous with
any project or undertaking, the termination of which The contracts of employment submitted by
was predetermined. respondents are highly suspect for not only being
ambiguous, but also for appearing to be tampered
Respondents asserted that petitioners were not with.
illegally dismissed, for their employment was
terminated due to the expiration of their terms of
employment. Petitioners contracts of employment
56. Agusan Del Norte Electric Cooperative vs.
with INNODATA were for a limited period only.
Cagampang ,G.R. No. 167627, October 10, 2008 -
There being no illegal dismissal, respondents
SAL
likewise maintained that petitioners were not
entitled to reinstatement and backwages. Facts: Joel Cagampang and Glenn Garzon started
working as linemen for Agusan del Norte Electric
ISSUE: Is the nature of employment of petitioners
Cooperative (ANECO) in 1990, under an employment
regular or fixed?
contract for a period not exceeding 3 months. They
RULING: Regular. The following employees are were required to work 8 hours a day and sometimes
accorded regular status: (1) those who are engaged on Sundays, with a daily salary of P122.00. When the
to perform activities which are necessary or contract expired, the two were laid-off for one to
desirable in the usual business or trade of the five days and then ordered to report back to work
employer, regardless of the length of their but on the basis of job orders. Their job orders, with
employment; and (2) those who were initially hired employment periods of about 3 months each, were
as casual employees, but have rendered at least one renewed several times, until 1998 & 1999 when such
year of service, whether continuous or broken, with contracts were no longer renewed, resulting in their
respect to the activity in which they are employed. loss of employment. In 2001, they filed a case for
illegal dismissal against ANECO, and prayed for
Undoubtedly, petitioners belong to the first type of payment of backwages, salary differential,
regular employees. Under Article 280 of the Labor allowances, premium for alleged work during
Code, the applicable test to determine whether an holidays and rest days, service incentive leave, and
employment should be considered regular or non- separation pay. The Labor Arbiter (LA) declared the
regular is the reasonable connection between the dismissal illegal and ordered ANECO to pay their
particular activity performed by the employee in money claims. The NLRC set aside the LA decision
relation to the usual business or trade of the except the portions granting service incentive leave
employer. pay; attorney's fees, fixed at ten percent (10%) of
the total money award to both respondents; and
In the case at bar, petitioners were employed by salary differential to Garzon. The CA reinstated the
INNODATA as formatters. The primary business of LA decision.
INNODATA is data encoding, and the formatting of
the data entered into the computers is an essential ISSUE: WON the respondents were illegally
part of the process of data encoding. Formatting dismissed
organizes the data encoded, making it easier to
understand for the clients and/or the intended end RULING: Yes. The test to determine whether
users thereof. Undeniably, the work performed by employment is regular or not is the reasonable
petitioners was necessary or desirable in the connection between the particular activity
business or trade of INNODATA. performed by the employee in relation to the usual
business or trade of the employer. Also, if the
employee has been performing the job for at least Trinidad, whose employments had to be co-
one year, even if the performance is not continuous terminous with the completion of specific company
or merely intermittent, the law deems the repeated projects. For this reason, every time the company
and continuing need for its performance as sufficient employed Trinidad, he had to execute an
evidence of the necessity, if not indispensability of employment contract with it, called Appointment as
that activity to the business. Thus, we held that Project Worker. Petitioner company stressed that
where the employment of project employees is employment intervals or gaps were inherent in the
extended long after the supposed project has been construction business. Petitioner company
finished, the employees are removed from the scope submitted an establishment termination report to
of project employees and are considered regular DOLE. The Labor Arbiter dismissed the complaint.
employees. The Labor Arbiter, however, ordered petitioner
company to pay Trinidad P1,500.00 in unpaid service
Respondents in the present case being regular incentive leave, taking into consideration the three-
employees, ANECO as the employer had the burden year prescriptive period for money claims, holding
of proof to show that the respondents' termination that, since Trinidad was a project employee and
was for a just cause. Unfortunately, however, what since his company submitted the appropriate
petitioners did was merely to refuse, without establishment termination report to DOLE, his loss of
justifiable reason, to renew respondents' work work cannot be regarded as unjust dismissal. NLRC
contracts for the performance of what would affirmed the Labor Arbiters ruling. The CA reversed
otherwise be regular jobs in relation to the trade or the NLRCs findings.
business of the former. Such conduct dismally falls
short of the requirements of our labor laws Issue: Does petitioner companys repeated rehiring
regarding dismissals. No twin notices of termination of respondent as project employee makes him a
were issued to the employees; hence, the employer regular employee?
did not observe due process in dismissing them from
their employment. Their dismissals were patently Ruling: No. The test for distinguishing a project
illegal. employee from a regular employee is whether or
not he has been assigned to carry out a specific
project or undertaking, with the duration and scope
of his engagement specified at the time his service is
57. G.R. No. 183250 March 10, 2010 contracted. Here, it is not disputed that petitioner
WILLIAM UY CONSTRUCTION CORP. and/or company contracted respondent Trinidads service
TERESITA UY and WILLIAM UY, Petitioners, by specific projects with the duration of his work
vs. JORGE R. TRINIDAD, Respondent.- CEL clearly set out in his employment contracts. He
remained a project employee regardless of the
Facts: Respondent Jorge R. Trinidad filed a complaint number of years and the various projects he worked
for illegal dismissal and unpaid benefits against for the company. Generally, length of service
petitioner company claiming that he had been provides a fair yardstick for determining when an
working with the latter company for 16 years since employee initially hired on a temporary basis
1988 as driver of its service vehicle, dump truck, and becomes a permanent one, entitled to the security
transit mixer. He had signed several employment and benefits of regularization. But this standard will
contracts which identified him as a project employee not be fair, if applied to the construction industry,
although he had always been assigned to work on simply because construction firms cannot guarantee
one project after another with some intervals. work and funding for its payrolls beyond the life of
Respondent further alleged that petitioner company each project. And getting projects is not a matter of
terminated him from work after it shut down course. Construction companies have no control
operations because of lack of projects. He learned over the decisions and resources of project
later, however, that although it opened up a project proponents or owners. There is no construction
in Batangas, it did not hire him back for that project. company that does not wish it has such control but
Petitioner company countered that it was in the the reality, understood by construction workers, is
construction business and that by the nature of such that work depended on decisions and developments
business, it had to hire and engage the services of over which construction companies have no say.
project construction workers, including respondent Respondent Trinidads series of employments with
petitioner company were co-terminous with its them their holiday pay, premium pay for holiday,
projects. When its Boni Serrano-Katipunan rest day, service incentive leave pay, and 13th
Interchange Project was finished, Trinidads month pay during the existence and duration of their
employment ended with it. He was not dismissed. employment. They also averred that they were not
His employment contract simply ended with the provided with sick and vacation leaves.
project for which he had signed up. His employment
history belies the claim that he continuously worked 59. Millennium Erectors Corp v Magallanes G.R. No.
for the company. Intervals or gaps separated one 184362 (2010)-JAY
contract from another. Petitioner company needed
only to show the last status of Trinidads Issue: Whether respondent has attained the status
employment, namely, that of a project employee of a regular employee?
under a contract that had ended and the companys
Facts: Respondent started working in 1988 as a
compliance with the reporting requirement for the
utility man for Millennium Erectors Corporation
termination of that employment. Indeed, both the
(petitioner) and Kenneth Construction
Labor Arbiter and the NLRC were satisfied that the
Corporation. He was assigned to different
fact of petitioner companys compliance with DOLE
construction projects undertaken by petitioner in
Order 19 had been proved in this case. Court
Metro Manila, the last of which was for a building in
REINSTATES the decision of NLRC.
Libis, Quezon City. In July of 2004 he was told not to
58. DACUITAL, ET AL vs. LM CAMUS ENGINEERING report for work anymore allegedly due to old age,
CORP- Toni prompting him to file on August 6, 2004 an illegal
dismissal complaint before the Labor Arbiter.
Facts: L.M. Camus Engineering Corporation (LMCEC)
is a domestic corporation engaged in construction, Petitioner claimed that respondent was a project
engineering, and air-conditioning business and Luis employee whom it hired for a building project in
M. Camus (Camus) is the company president. Judy O. Libis on January 30, 2003, to prove which it
Dacuital (Dacuital), Eugenio L. Mondano, Jr., Joseph submitted the employment contract signed by
Galer (Galer), Mariano Morales, Roberto Ruance him; that on August 3, 2004, respondents services
(Ruance), Joseph Porcadilla, Raulito Palad (Palad), were terminated as the project was nearing
Ricardo Digamon (Digamon), Nonito Prisco, Eulogio completion; and he was given financial assistance in
M. Tutor, Melvin Pepito, Helyto N. Reyes (Reyes), the amount of P2,000, for which he signed a
Randolf C. Baludo (Baludo), Alberto Epondol, Rodelo quitclaim and waiver. Petitioner likewise submitted a
A. Susper, Evaristo Vigori, Jonathan P. Ayaay, Felipe termination report to the Department of Labor and
Erilla, Aris A. Garcia (Aris), Roy A. Garcia (Roy), and Employment (DOLE) dated August 17, 2004.
Restituto Tapanan (Tapanan) were hired by LMCEC
Labor Arbiter ruled in favor of petitioner and
as welder, tinsmith, pipefitter, and mechanical
dismissed the complaint, holding that respondent
employees.
knew of the nature of his employment as a project
During the months of January, February and March employee. NLRC set aside the Arbiters decision
2001, Dacuital, et al were required by LMCEC to holding that respondent was a regular employee,
surrender their identification cards and ATM cards not a project employee. CA affirmed. MR was
and were ordered to execute contracts of denied, hence this case.
employment. Most of the petitioners did not comply
Ruling: The Court finds that, indeed, respondent was
with the directive as they believed that it was only
a regular, not a project employee.
LMs strategy to get rid of petitioners regular status
since they would become new employees Saberola v. Suarez reiterates the well-settled
disregarding their length of service. Petitioners were definition of project employee, viz:
later dismissed from employment.
A project employee is one whose "employment has
The complaint for illegal dismissal and non-payment been fixed for a specific project or undertaking, the
of monetary benefits was filed and the employees completion or termination of which has been
alleged that they were illegally dismissed from determined at the time of the engagement of the
employment and that their employer failed to pay employee or where the work or service to be
performed is seasonal in nature and the was assigned Pacific Plaza Towers. Later, the five
employment is for the duration of the season." filed a complaint for illegal dismissal and non-
(emphasis and underscoring supplied) payment of holiday pay, service incentive leave pay,
13th month pay, and night shift differential pay.
And Equipment Technical Services v. Court of Exodus denied the allegations and contended that
Appeals emphasizes the difference between a Gregorio absented himself from work and applied as
regular employee and a project employee: a painter with SAEI-EEI which is the general building
contractor of Pacific plaza Towers. Since then, he
As the Court has consistently held, the service never reported back to work; Guillermo absented
of project employees are coterminus with the himself from work without leave. When he reported
project and may be terminated upon the end or for work the following day, he was reprimanded for
completion of that project or project phase for being aabsent without leave and thereafter was
which they were hired. Regular employees, in unheard of after until the filing of this complaint,
contrast, enjoy security of tenure and are entitled to and Fernando, Ferdinand, and Miguel were caught
hold on to their work or position until their services eating during working hours for which they were
are terminated by any of the modes recognized reprimanded by their foreman. Since then, they no
under the Labor Code. (emphasis and underscoring longer reported for work.
supplied)
Issue: WON the respondents are non-project
Petitioners various payrolls dating as early as 2001 employees.
show that respondent had been employed by it. As
aptly observed by the appellate court, these Ruling: there are two types of employees in the
documents, rather than sustaining petitioners construction industry. The first is project employees
argument, only serve to support respondents or those employed in connection with a particular
contention that he had been employed in various construction project and such employment is
projects, if not for 16 years, at the very least two coterminous with each project to which they were
years prior to his dismissal. assigned. The second is non-project employee or
those employed without reference to any project.
Assuming arguendo that petitioner hired The second category is where respondents are
respondent initially on a per project basis, his classified. As such, they are regular employees. It is
continued rehiring, as shown by the sample payrolls clear that when one project is completed,
converted his status to that of a regular respondents were automatically transferred to
employee. Following Cocomangas Beach Hotel another project.
Resort v. Visca, the repeated and continuing need
for respondents services is sufficient evidence of the
necessity, if not indispensability, of his services to
petitioner's business and, as a regular employee, he 61. Leyte Geothermal vs PNOC-Juneey
could only be dismissed from employment for a just
or authorized cause. FACTS: Respondent is a GOCC while petitioner is a
legitimate labor organization. Among respondents
geothermal projects is the Leyte Geothermal Power
Project located at the Greater Tongonan Geothermal
60.EXODUS VS BISCOCHO- Julius Reservation in Leyte. Thus, the respondent hired and
employed hundreds of employees on a contractual
Facts: Exodus is a duly licensed labor contractor for basis, whereby, their employment was only good up
the painting of residential houses, condominium to the completion or termination of the project and
units and commercial buildings. Exodus obtained would automatically expire upon the completion of
from Dutch Boy a contract for the painting of the such project. Majority of the employees hired by
Imperial Sky Garden. Dutch Boy awarded another respondent in its Leyte Geothermal Power Projects
contract to Exodus for the painting of Pacific Plaza had become members of petitioner. In view of that
Towers. Exodus hired Biscocho, Pereda, Mariano and circumstance, the petitioner demands from the
were assigned at the Imperial Sky Garden, Bellita respondent for recognition of it as the collective
was assigned to work at the house of Yap in Ayala, bargaining agent of said employees and for a CBA
Alabang,, and Bobillo, who, together with the four,
negotiation with it. However, the respondent did not employment involving a specific undertaking which
heed such demands of the petitioner. Sometime in completion or termination has been determined at
1998 when the project was about to be completed, the time of the particular employees engagement.
the [respondent] proceeded to serve Notices of
Termination of Employment upon the employees
who are members of the petitioner. On December
28, 1998, the petitioner filed a Notice of Strike with 62. St. Paul College Quezon City vs Ancheta-
DOLE against the [respondent] on the ground of Mamugay
purported commission by the latter of unfair labor
Facts: Remigio Ancheta was a full-time
practice for refusal to bargain collectively, union
probationary teacher in the School Year 1996-1997
busting and mass termination. On the same day,
which was renewed in the following SY 1997-1998.
the petitioner declared a strike and staged such
His wife, Cynthia was hired as a part time teacher of
strike. Secretary of Labor intervened and ordered all
the Mass Communication Department in the second
workers to return to work. However, petitioner did
semester of SY 1996-1997 and her appointment was
not abide. NLRC: ruled that the employees are
renewed for SY 1997-1998. On February 13, 1998,
PROJECT EMPLOYEES, and the strike as ILLEGAL
respondents signified their intentions to renew their
Petitioner Union contends that its officers and
contracts for SY 1998-1999.
members performed activities that were usually
necessary and desirable to respondents usual They were later sent two letters informing them that
business. the school is extending to them new contracts for SY
1998-1999. A letter was written to Remigio Michael,
ISSUE: WON they are project employees
enumerating the departmental and instructional
HELD: They are PROJECT EMPLOYEES policies that spouses failed to comply with, such as
the late submission of final grades, failure to submit
By entering into such a contract, an employee is final test questions to the Program Coordinator, the
deemed to understand that his employment is giving of tests in the essay form instead of the
coterminous with the project. He may not expect to multiple choice format as mandated by the school,
be employed continuously beyond the completion of failure to report to work on time; the high number of
the project. It is of judicial notice that project students with failing grades in the classes that they
employees engaged for manual services or those for handled, and not being open to suggestions to
special skills like those of carpenters or masons, are, improve themselves as teachers, among others. Sr.
as a rule, unschooled. However, this fact alone is not Bernadette (Department Coordinator) endorsed the
a valid reason for bestowing special treatment on immediate termination of the teaching services of
them or for invalidating a contract of employment. the spouses.
Project employment contracts are not lopsided
agreements in favor of only one party thereto. The Respondent spouses were given an opportunity to
employers interest is equally important as that of comment on the letter-recommendation.
the employees for theirs is the interest that propels Subsequently however, they received their
economic activity. While it may be true that it is the respective letters of termination. Thus, spouses filed
employer who drafts project employment contracts a Complaint for illegal dismissal. St. Paul contends
with its business interest as overriding consideration, that it did not extend the contracts of respondent
such contracts do not, of necessity, prejudice the spouses. Although, it has sent letters to the spouses
employee. Neither is the employee left helpless by a informing them that the school is extending to them
prejudicial employment contract. After all, under the new contracts for the coming school year, the letters
law, the interest of the worker is paramount. Unions do not constitute as actual employment contracts
own admission, both parties had executed the but merely offers to teach on the said school year.
contracts freely and voluntarily without force, duress
Issue: Were the spouses Anchetta illegally
or acts tending to vitiate the worker[s] consent.
dismissed?
Thus, we see no reason not to honor and give effect
to the terms and conditions stipulated therein. The
test to determine whether an individual is a project
employee lies in setting a fixed period of
Ruling: No. It is important that the contract of Schools cannot be required to adopt standards
probationary employment specify the period or term which barely satisfy criteria set for government
of its effectivity. recognition. The same academic freedom grants the
school the autonomy to decide for itself the terms
Section 91 of the Manual of Regulations for Private and conditions for hiring its teacher, subject of
Schools, states that: course to the overarching limitations under the
Labor Code. The authority to hire is likewise covered
Section 91.Employment Contract. Every contract and protected by its management prerogative the
of employment shall specify the designation, right of an employer to regulate all aspects of
qualification, salary rate, the period and nature of employment, such as hiring, the freedom to
service and its date of effectivity, and such other prescribe work assignments, working methods,
terms and condition of employment as may be process to be followed, regulation regarding transfer
consistent with laws and rules, regulations and of employees, supervision of their work, lay-off and
standards of the school. A copy of the contract shall discipline, and dismissal and recall of workers.
be furnished the personnel concerned.

The failure to stipulate its precise duration could


lead to the inference that the contract is binding for 63. Lynvil Fishing Enterprises vs. Ariola, G.R. No.
the full three-year probationary period. Therefore, 181974, February 1, 2012-June
the letters sent by petitioner Sr. Bernadette, which
were void of any specifics cannot be considered as Facts: Petitioner Lynvil Fishing Enterprises, Inc.
contracts. The closest they can resemble to are that (Lynvil) is engaged in deep-sea fishing. Respondents
of informal correspondence among the said services were engaged in various capacities: Andres
individuals. As such, petitioner school has the right G. Ariola, captain; Jessie D. Alcovendas, chief mate;
not to renew the contracts of the respondents, the Jimmy B. Calinao, chief engineer; Ismael G. Nubla,
old ones having been expired at the end of their cook; Elorde Baez, oiler; and Leopoldo G. Sebullen,
terms. bodegero.

Even assuming that the employment contracts On Aug. 1, 1998, Lynvil received a report from
between the school and the spouses were renewed, Ramonito Clarido, one of its employees, that on July
there was a valid and just cause for their dismissal. 31, 1998, he witnessed that while on board the
The Labor Code commands that before an employer company vessel Analyn VIII, respondents conspired
may legally dismiss an employee from the service, with one another and stole eight tubs of pampano
the requirement of substantial and procedural due and tangigue fish and delivered them to another
process must be complied with. Under the vessel.
requirement of substantial due process, the grounds
for termination of employment must be based on Petitioner filed a criminal complaint against
just or authorized causes. respondents before the office of the City Prosecutor
of Malabon City which found probable cause for
The plain admissions of the charges against them indictment of respondents for the crime of qualified
were the considerations taken into account by the theft. Relying on the finding and Nasipit Lumber
petitioner school in their decision not to renew the Company v. NLRC, 257 Phil. 937 (1989), Lynvil
respondent spouses' employment contracts. This is a asserted there was sufficient basis for valid
right of the school that is mandated by law and termination of employment of respondents based
jurisprudence. It is the prerogative of the school to on serious misconduct and/or loss of trust and
set high standards of efficiency for its teachers since confidence.
quality education is a mandate of the Constitution.
As long as the standards fixed are reasonable and Issue: 1) WON Ariola et.al were illegally dismissed?
not arbitrary, courts are not at liberty to set them
aside. 2) WON Lynvil complied with the procedural
requirement of due process in Section 2, Rule XXIII,
Book V of the Rules Implementing the Labor Code in
its dismissal of Ariola et. al?
Ruling: 1)No. The Court ruled that there was valid days why they should not be dismissed from the
cause for respondents dismissal. In illegal dismissal service. Alcovendas was the only one who signed the
cases, the employer bears the burden of proving receipt of the notice. The others, as claimed by
that the termination was for a valid or authorized Lynvil, refused to sign. The other employees argue
cause. that no notice was given to them. Despite the
inconsistencies, what is clear is that no final written
Just cause is required for a valid dismissal. The Labor notice or notices of termination were sent to the
Code provides that an employer may terminate an employees.
employment based on fraud or willful breach of the
trust reposed on the employee. Such breach is The twin requirements of notice and hearing
considered willful if it is done intentionally, constitute the elements of due process in cases of
knowingly, and purposely, without justifiable excuse, employee's dismissal. The requirement of notice is
as distinguished from an act done carelessly, intended to inform the employee concerned of the
thoughtlessly, heedlessly or inadvertently. It must employer's intent to dismiss and the reason for the
also be based on substantial evidence and not on the proposed dismissal. Upon the other hand, the
employers whims or caprices or suspicions requirement of hearing affords the employee an
otherwise, the employee would eternally remain at opportunity to answer his employer's charges
the mercy of the employer. Loss of confidence must against him and accordingly, to defend himself
not be indiscriminately used as a shield by the therefrom before dismissal is effected. Obviously,
employer against a claim that the dismissal of an the second written notice, as indispensable as the
employee was arbitrary. And, in order to constitute a first, is intended to ensure the observance of due
just cause for dismissal, the act complained of must process.
be work-related and shows that the employee
concerned is unfit to continue working for the Applying the rule to the facts at hand, the Court
employer. In addition, loss of confidence as a just granted a monetary award of P50,000.00 as nominal
cause for termination of employment is premised on damages, pursuant to the fresh ruling of the Court
the fact that the employee concerned holds a in Culili v. Eastern Communication Philippines,
position of responsibility, trust and confidence or Inc. Due to the failure of Lynvil to follow the
that the employee concerned is entrusted with procedural requirement of two-notice rule, nominal
confidence with respect to delicate matters, such as damages are due to respondents despite their
the handling or care and protection of the property dismissal for just cause.
and assets of the employer. The betrayal of this trust
is the essence of the offense for which an employee
is penalized.
64. D.M. Consunji Inc. vs. Jamin, G.R. No. 192514,
Breach of trust is present in this case.(in arriving at April 18, 2012 (citing Maraguinot)- krizza
this conclusion, the court took into account the
Facts: On December 17, 1968, petitioner D.M.
positive and clear narration of facts of the three
Consunji, Inc. (DMCI), a construction company, hired
witnesses to the commission of qualified theft.)
respondent Estelito L. Jamin as a laborer. Sometime
2.) No. It is required that the employer furnish the in 1975, Jamin became a helper carpenter. Since his
employee with two written notices: (1) a written initial hiring, Jamins employment contract had been
notice served on the employee specifying the ground renewed a number of times. On March 20, 1999, his
or grounds for termination, and giving to said work at DMCI was terminated due to the completion
employee reasonable opportunity within which to of the SM Manila project. This termination marked
explain his side; and (2) a written notice of the end of his employment with DMCI as he was not
termination served on the employee indicating that rehired again. On April 5, 1999, Jamin filed a
upon due consideration of all the circumstances, complaint for illegal dismissal, with several money
grounds have been established to justify his claims (including attorneys fees), against DMCI and
termination. its President/General Manager, David M. Consunji.
Jamin alleged that DMCI terminated his employment
From the records, there was only one written notice without a just and authorized cause at a time when
which required respondents to explain within five (5) he was already 55 years old and had no independent
source of livelihood. He claimed that he rendered almost four years in his employment for the period
service to DMCI continuously for almost 31 years. In between July 28, 1980 (the supposed completion
addition to the schedule of projects (where he was date of the Midtown Plaza project) and June 13,
assigned) submitted by DMCI to the labor arbiter, he 1984 (the start of the IRRI Dorm IV project), the gap
alleged that he worked for three other DMCI was caused by the companys omission of the three
projects: Twin Towers, Ritz Towers, from July 29, projects.
1980 to June 12, 1982; New Istana Project, B.S.B.
Brunei, from June 23, 1982 to February 16, 1984; Jamins employment history with DMCI stands out
and New Istana Project, B.S.B. Brunei, from January for his continuous, repeated and successive rehiring
24, 1986 to May 25, 1986. DMCI denied liability. It in the companys construction projects. In all the 38
argued that it hired Jamin on a project-to-project projects where DMCI engaged Jamins services, the
basis, from the start of his engagement in 1968 until tasks he performed as a carpenter were indisputably
the completion of its SM Manila project on March necessary and desirable in DMCIs construction
20, 1999 where Jamin last worked. With the business. He might not have been a member of a
completion of the project, it terminated Jamins work pool as DMCI insisted that it does not maintain
employment. It alleged that it submitted a report to a work pool, but his continuous rehiring and the
the Department of Labor and Employment (DOLE) nature of his work unmistakably made him a regular
everytime it terminated Jamins services. employee. In Maraguinot, Jr. v. NLRC, the Court held
that once a project or work pool employee has been:
ISSUE: Whether there was violation of security of (1) continuously, as opposed to intermittently,
tenure? rehired by the same employer for the same tasks or
nature of tasks; and (2) these tasks are vital,
RULING: YES. Jamin worked for DMCI for almost 31 necessary and indispensable to the usual business or
years, initially as a laborer and, for the most part, as trade of the employer, then the employee must be
a carpenter. Through all those years, DMCI treated deemed a regular employee.
him as a project employee, so that he never
obtained tenure. On the surface and at first glance,
DMCI appears to be correct. Jamin entered into a
contract of employment (actually an appointment 65. ANTONIO E. UNICA vs. ANSCOR SWIRE SHIP
paper to which he signified his conformity) with MANAGEMENT CORPORATION,February 12, 2014-
DMCI either as a field worker, a temporary worker, a Duron
casual employee, or a project employee every time
DMCI needed his services and a termination of FACTS: Since the late 1980s, petitioner Antonio
employment paper was served on him upon Unica was employed by respondent Anscor Swire
completion of every project or phase of the project Management Corp (a manning agency) under
where he worked. The CA pierced the cover of various contracts. In his last contract, petitioner was
Jamins project employment contract and declared deployed for a period of 9 months, from January 29,
him a regular employee who had been dismissed 2000 to October 25, 2000. Since the vessel was at
without cause and without notice. To reiterate, the sea, petitioner was only repatriated on November
CAs findings were based on: (1) Jamins repeated 14, 2000, or 20 days after the expiration of his
and successive engagements in DMCIs construction contract of employment. Petitioner claims that since
projects, and (2) Jamins performance of activities he was allowed to stay in the vessel for another 20
necessary or desirable in DMCIs usual trade or days, there was an implied renewal of his
business. employment contract. Hence, when he was
repatriated on November 14, 2000 without a valid
We reviewed Jamins employment contracts as the cause, he was illegally dismissed. Petitioner filed a
CA did and we noted that while the contracts indeed case against the respondent for illegal dismissal,
show that Jamin had been engaged as a project payment of retirement, disability and medical
employee, there was an almost unbroken string of benefits, separation and holiday pay. Respondent, in
Jamins rehiring from December 17, 1968 up to the its defense, argues that petitioner was hired for a
termination of his employment on March 20, 1999. fixed period, the duration of which depends upon
While the history of Jamins employment (schedule the mutual agreement of the parties. Petitioners
of projects) relied upon by DMCI shows a gap of employment was, therefore, co-terminus with the
term of his contract. Hence when he was Thus, when petitioner's contract ended on October
repatriated, the term of his contract was completed. 25, 2000, his employment is deemed automatically
terminated, there being no mutually-agreed renewal
LA ruled in petitioner's favor. It held that since or extension of the expired contract. However,
petitioner was not repatriated at the expiration of petitioner is entitled to be paid his wages after the
his contract on October 25 and was allowed to expiration of his contract until the vessel's arrival at
continue working on board respondent's vessel until a convenient port. Section 19 of the Standard Terms
November 14, his contract with respondent was and Conditions Governing the Employment of
impliedly renewed for another 9 months. Filipino Seafarers On-Board Ocean-Going Vessels is
clear on this point:
NRLC affirmed LA's decision with modification. NLRC
ruled that the contract did not expire on October 25, REPATRIATION. A. If the vessel is outside the
2000, but was impliedly extended for another nine Philippines upon the expiration of the contract, the
months. This is because it was only on November 14, seafarer shall continue his service on board until the
2000 when petitioner was told by respondent to vessel's arrival at a convenient port and/or after
disembark because he would be repatriated. Since arrival of the replacement crew, provided that, in
there was an implied extension of the contract for any case, the continuance of such service shall not
another nine months, petitioner is, therefore, exceed three months. The seafarer shall be entitled
entitled to payment of the unexpired term of his to earned wages and benefits as provided in his
implied contract. contract.

CA set aside the decision of the NLRC. The CA ruled CA affirmed. Petition denied. Respondent is
that there was no implied renewal of contract and DIRECTED to PAY petitioner his salary from October
the 20 days extension was due to the fact that the 26, 2000 until November 14, 2000.
ship was still at sea.

ISSUE: WON there was an implied renewal of


Petitioner's contract of employment with 66. IMASEN PHILIPPINE MANUFACTURING
respondent CORPORATION v. ALCON-Karen

RULING: No implied renewal. Petitioner's late Facts: Imasen is a domestic corporation engaged in
disembarkation was not without valid reason (note: the manufacture of auto seat-recliners and slide-
employment contract ended on Oct. 25 2000, adjusters. It hired the respondents as manual
disembarked on November 14 2000). Respondent welders in 2001. On October 5, 2002, the
could not have disembarked petitioner on the date respondents reported for work on the second shift -
of the termination of his employment contract as from 8:00 pm to 5:00 am of the following day. At
the vessel was still in the middle of the sea. Clearly, around 12:40 am, Cyrus A. Altiche, Imasen's security
it was impossible for petitioner to safely disembark guard on duty, went to patrol and inspect the
immediately upon the expiration of his contract, production plant's premises. When Altiche reached
since he must disembark at a convenient port. Thus, Imasen's Press Area, he heard the sound of a
petitioner's stay in the vessel for another 20 days running industrial fan. Intending to turn the fan off,
should not be interpreted as an implied extension of he followed the sound that led him to the plant's
his contract. A seaman need not physically "Tool and Die" section. At the "Tool and Die" section,
disembark from a vessel at the expiration of his Altiche saw the respondents having sexual
employment contract to have such contract intercourse on the floor, using a piece of carton as
considered terminated. mattress. Altiche immediately went back to the
guard house and relayed what he saw to Ogana,
As a rule, seafarers are considered contractual another security guard on duty. Ogana made a
employees. Their employment is governed by the follow-up inspection. Ogana went to the "Tool and
contracts they sign everytime they are rehired and Die" section and saw several employees, including
their employment is terminated when the contract the respondents, already leaving the area. He
expires. Their employment is contractually fixed for noticed, however, that Alcon picked up the carton
a certain period of time. that Altiche claimed the respondents used as
mattress during their sexual act, and returned it to adults belong, as a principled ideal, to the realm of
the place where the cartons were kept. Altiche then purely private relations. Whether aroused by lust or
submitted a handwritten report of the incident to inflamed by sincere affection, sexual acts should be
Imasen's Finance and Administration Manager. carried out at such place, time and circumstance
that, by the generally accepted norms of conduct,
On October 14, 2002, Imasen issued the respondents will not offend public decency nor disturb the
separate interoffice memoranda informing them of generally held or accepted social morals. Under
Altiche's report on the October 5, 2002 incident and these parameters, sexual acts between two
directing them to submit their individual consenting adults do not have a place in the work
explanation. The respondents complied with the environment.
directive; they claimed that they were merely
sleeping in the "Tool and Die" section at the time of Indisputably, the respondents engaged in sexual
the incident. They also claimed that other employees intercourse inside company premises and during
were near the area, making the commission of the work hours. These circumstances, by themselves, are
act charged impossible. already punishable misconduct. Added to these
considerations, however, is the implication that the
On October 22, 2002, Imasen issued the respondents respondents did not only disregard company rules
another interoffice memorandum directing them to but flaunted their disregard in a manner that could
appear at the formal hearing of the administrative reflect adversely on the status of ethics and morality
charge against them. The hearing was conducted on in the company.
October 30, 2002, presided by a mediator and
attended by the representatives of Imasen, the Additionally, the respondents engaged in sexual
respondents, Altiche and Ogana. Altiche and Ogana intercourse in an area where co-employees or other
reiterated the narrations in Altiche's handwritten company personnel have ready and available access.
report. The respondents likewise committed their act at a
time when the employees were expected to be and
On December 4, 2002, Imasen issued the had, in fact, been at their respective posts, and when
respondents separate interoffice they themselves were supposed to be, as all other
memoranda terminating their services. It found the employees had in fact been, working.
respondents guilty of the act charged which it
considered as "gross misconduct contrary to the Under these factual premises and in the context of
existing policies, rules and regulations of the legal parameters we discussed, we cannot help but
company." consider the respondents' misconduct to be of grave
and aggravated character so that the company was
ISSUE: WON the act of engaging in sexual justified in imposing the highest penalty available
intercourse inside company premises during work dismissal. Their infraction transgressed the bounds
hours is serious misconduct? of socially and morally accepted human public
behavior, and at the same time showed brazen
disregard for the respect that their employer
HELD: Yes. Misconduct is defined as an improper or expected of them as employees. By their
wrong conduct. It is a transgression of some misconduct, the respondents, in effect, issued an
established and definite rule of action, a forbidden open invitation for others to commit the same
act, a dereliction of duty, willful in character, and infraction, with like disregard for their employer's
implies wrongful intent and not mere error in rules, for the respect owed to their employer, and
judgment. For misconduct or improper behavior to for their co-employees' sensitivities. Taken together,
be a just cause for dismissal, the following elements these considerations reveal a depraved disposition
must concur: (a) the misconduct must be serious; (b) that the Court cannot but consider as a valid cause
it must relate to the performance of the employee's for dismissal.
duties showing that the employee has become unfit
to continue working for the employer; and (c) it
must have been performed with wrongful intent.

Sexual acts and intimacies between two consenting


67. Cheryll Santos Leus vs, St. Scholasticas College, against either the father or the mother. In such a
G.R. No. 187226, January 28, 2015- Christal case, the disgraceful and immoral conduct consists
of having extramarital relations with a married
Facts: Cheryll was a non-teaching personnel at St. person. The sanctity of marriage is constitutionally
Scholaticas College, a Catholic School. They adapt recognized and likewise affirmed by our statutes as a
the teaching of the Roman Catholic faith. Cheryll special contract of permanent union. Accordingly,
became pregnant out of wedlock with her boyfriend. judicial employees have been sanctioned for their
The school dismissed her since under the 1992 dalliances with married persons or for their own
Manual for Regulation of Public Schools (1992 betrayals of the marital vow of fidelity. In this case, it
MRPS), one of the grounds for dismissal is was not disputed that, like respondent, the father of
disgraceful or immoral conduct. her child was unmarried. Therefore, respondent
cannot be held liable for disgraceful and immoral
Issue: May an employer dismiss an employee on the conduct simply because she gave birth to the child
ground that the latter got pregnant out of wedlock? Christian Jeon out of wedlock.
(What if the employer is a strict Catholic school that
imposes strict religious regulations, will this alone
validate the dismissal made to the erring employee?)
68. DEOFERIO VS. INTEL TECH PHILIPPINES (R-U,
Ruling:No. Her pregnancy out of wedlock does not Glenna)
constitute a valid ground to terminate her
employment. Facts: Intel Technology Philippines, Inc. employed
Deoferio as a product quality and reliability
Disgraceful conduct is viewed in two ways, the engineer. Intel assigned him to the United States as
public and secular view and religious view. Our a validation engineer for an agreed period of two
laws concern the first view. The jurisdiction of the years. However, Deoferio was repatriated to the
court is limited to the first view. Disgraceful conduct Philippines after being confined at Providence St.
per se will not amount to violation of the law the Vincent Medical Center for major depression with
conduct must affect or poses a danger to the psychosis. In the Philippines, he worked as a product
conditions of society, for example, the sanctity of engineer. Deoferio underwent a series of medical
marriage, right to privacy and the like. If the Court and psychiatric treatment at Intels expense after his
will entertain the second view then it would limit confinement in the United States. He was diagnosed
religious freedom. by several physicians that suffering from mood
disorder, major depression and auditory
The Court cited Estrada vs. Escritur in the said case, hallucination. After several consultations, Dr. Lee
stating the following relevant explanation; issued a psychiatric report concluding and stating
that Deoferios psychotic symptoms are not curable
(1) if the father of the child is himself unmarried,
within a period of six months and will negatively
the woman is not ordinarily administratively liable
affect his work and social relation with his co-
for disgraceful and immoral conduct. It may be a
workers. Pursuant to these findings, Intel issued
not-so-ideal situation and may cause complications
Deoferio a notice of termination on March 10, 2006.
for both mother and child but it does not give cause
for administrative sanction. There is no law which Deoferio responded to his termination of
penalizes an unmarried mother under those employment by filing a complaint for illegal dismissal
circumstances by reason of her sexual conduct or with prayer for money claims against respondents
proscribes the consensual sexual activity between Intel and Mike Wentling (respondents). He denied
two unmarried persons. Neither does the situation that he ever had mental illness and insisted that he
contravene any fundamental state policy as satisfactorily performed his duties as product
expressed in the Constitution, a document that engineer. He argued that Intel violated his statutory
accommodates various belief systems irrespective of right to procedural due process when it summarily
dogmatic origins. issued a notice of termination.
(2) if the father of the child born out of wedlock is
himself married to a woman other than the mother,
then there is a cause for administrative sanction
In defense, the respondents argued that Deoferios CA affirmed NLRCs decision.
dismissal was based on Dr. Lees certification that:
(1) His schizophrenia was not curable within a period 1. Yes.
of six months even with proper medical treatment;
and (2) His continued employment would be The third element substantiates the contention that
prejudicial to his and to the other employees health. the employee has indeed been suffering from a
The respondents also insisted that Deoferios disease that: (1) is prejudicial to his health as well as
presence at Intels premises would pose an actual to the health of his co-employees; and (2) cannot be
harm to his co-employees as shown by his previous cured within a period of six months even with proper
acts. On May 8, 2003, Deoferio emailed an Intel medical treatment. Without the medical certificate,
employee with this message: All souls day back to there can be no authorized cause for the employees
work Monday WW45.1 On January 18, 2005, he cut dismissal. The absence of this element thus renders
the mouse cables, stepped on the keyboards, and the dismissal void and illegal.
disarranged the desks of his co-employees. The
The present case involves termination due to disease
respondents also highlighted that Deoferio incurred
an authorized cause for dismissal under Article 284
numerous absences from work due to his mental
of the Labor Code. As substantive requirements, the
condition, specifically, from January 31, 2002 until
Labor Code and its IRR33 require the presence of the
February 28, 2002, from August 2002 until
following elements:
September 2002, and from May 2003 until July 2003.
Deoferio also took an administrative leave with pay (1) An employer has been found to be suffering from
from January 2005 until December 2005. any disease.
The respondents further asserted that the twin- (2) His continued employment is prohibited by law
notice requirement in dismissals does not apply to or prejudicial to his health, as well as to the health of
terminations under Article 284 of the Labor Code. his co-employees.
They emphasized that the Labor Codes
implementing rules (IRR) only requires a competent (3) A competent public health authority certifies that
public health authoritys certification to effectively the disease is of such nature or at such a stage that it
terminate the services of an employee. cannot be cured within a period of six months even
with proper medical treatment. With respect to the
Issues: first and second elements, the Court liberally
construed the phrase "prejudicial to his health as
Whether Deoferio was suffering from schizophrenia
well as to the health of his co-employees" to mean
and whether his continued employment was
"prejudicial to his health or to the health of his co-
prejudicial to his health, as well as to the health of
employees." We did not limit the scope of this
his co-employees;
phrase to contagious diseases for the reason that
Whether the twin notice requirement in dismissals this phrase is preceded by the phrase "any disease"
applies to terminations due to disease; and under Article 284 of the Labor Code, to wit:

Whether Deoferio is entitled to salary differential, Art. 284. Disease as ground for termination. An
backwages, separation pay, moral and exemplary employer may terminate the services of an
damages, as well as attorneys fees. employee who has been found to be suffering from
any disease and whose continued employment is
Ruling: prohibited by law or is prejudicial to his health as
well as to the health of his co-employees: Provided,
Labor Arbiter ruled that Deoferio had been validly That he is paid separation pay equivalent to at least
dimissed. one (1) month salary or to one-half (1/2) month
salary for every year of service, whichever is greater,
a fraction of at least six (6) months being considered
as one (1) whole year.
NLRC affirmed LAs ruling.
2. Yes. Section 2, Rule 1, Book VI of the IRR expressly 69. EMERITUS CASE- KATE
states that the employee should be afforded
procedural due process in all cases of dismissals. Facts: In August 2000, petitioner hired respondent as
one of its security guards. During his employment,
3. Yes. respondent was assigned to petitioner's various
clients. On 10 December 2005, respondent was
Intels violation of Deoferios right to statutory relieved from his post.On 27 January 2006,
procedural due process warrants the payment of respondent filed a complaint for underpayment of
indemnity in the form of nominal damages. In Jaka wages, non-payment of legal and special holiday
Food Processing Corp. v. Pacot,41 we distinguished pay, premium pay for rest day and underpayment of
between terminations based on Article 282 of the ECOLA before the Department of Labor and
Labor Code42 and dismissals under Article 283 of the Employment, National Capital Region. The hearing
Labor Code.43 We then pegged the nominal officer recommended the dismissal of the complaint
damages at P30,000.00 if the dismissal is based on a since the claims were already paid.
just cause but the employer failed to comply with
the twin-notice requirement. On the other hand, we On 16 June 2006, respondent filed a complaint for
fixed the nominal damages at P50,000.00 if the illegal dismissal and payment of separation pay
dismissal is due to an authorized cause under Article against petitioner before the Conciliation and
283 of the Labor Code but the employer failed to Mediation Center of the NLRC. On 14 July 2006,
comply with the notice requirement. The reason is respondent filed another complaint for illegal
that dismissals for just cause imply that the dismissal, underpayment of salaries and non-
employee has committed a violation against the payment of full backwages before the NLRC.
employer, while terminations under Article 283 of
the Labor Code are initiated by the employer in the Respondent claimed that on various dates in
exercise of his management prerogative. December 2005 and from January to May 2006,4 he
went to petitioners office to follow-up his next
With respect to Article 284 of the Labor Code, assignment. After more than six months since his
terminations due to disease do not entail any last assignment, still respondent was not given a
wrongdoing on the part of the employee. It also new assignment. Respondent argued that if an
does not purely involve the employers willful and employee is on floating status for more than six
voluntary exercise of management prerogative a months, such employee is deemed illegally
function associated with the employer's inherent dismissed.
right to control and effectively manage its
enterprise.44 Rather, terminations due to disease Petitioner denied dismissing respondent. Petitioner
are occasioned by matters generally beyond the admitted that it relieved respondent from his last
worker and the employer's control. assignment and petitioner required respondent to
report to the head office within 48 hours from
In fixing the amount of nominal damages whose receipt of the order of relief. Respondent allegedly
determination is addressed to our sound discretion, failed to comply. Petitioner also claimed that there
the Court should take into account several factors was no showing that respondent was prevented
surrounding the case, such as: (1) the employers from returning to his work..
financial, medical, and/or moral assistance to the
sick employee; (2) the flexibility and leeway that the Issue: W/N respondent was illegally dismissed?
employer allowed the sick employee in performing
his duties while attending to his medical needs; (3) Ruling:
the employers grant of other termination benefits in
Yes. Petitioner admits relieving respondent from his
favor of the employee; and (4) whether there was a
post as security guard on 10 December 2005. There
bona fide attempt on the part of the employer to
is also no dispute that respondent remained on
comply with the twin-notice requirement as
floating status at the time he filed his complaint for
opposed to giving no notice at all.
illegal dismissal on 16 June 2006. In other words,
respondent was on floating status from 10
December 2005 to 16 June 2006 or more than six
months. Petitioners allegation of sending
respondent a notice sometime in January 2006, respondent points out that he was not reinstated by
requiring him to report for work, is unsubstantiated, petitioner Emeritus Security and Maintenance
and thus, self-serving. Systems, Inc. but was employed by another
company, Emme Security and Maintenance Systems,
The Court agrees with the ruling of the Labor Arbiter, Inc. (Emme). Thus, according to respondent, he was
NLRC and Court of Appeals that a floating status of a not reinstated at all.
security guard, such as respondent, for more than six
months constitutes constructive dismissal. In Petitioner counters that Emeritus and Emme are
Nationwide Security and Allied Services, Inc. v. sister companies with the same Board of Directors
Valderama,8 the Court held: and officers, arguing that Emeritus and Emme are in
effect one and the same corporation.
x x x the temporary inactivity or "floating status" of
security guards should continue only for six months. Considering petitioner's undisputed claim that
Otherwise, the security agency concerned could be Emeritus and Emme are one and the same, there is
liable for constructive dismissal. The failure of no basis in respondent's allegation that he was not
petitioner to give respondent a work assignment reinstated to his previous employment. Besides,
beyond the reasonable six-month period makes it respondent assails the corporate personalities of
liable for constructive dismissal. Emeritus and Emme only in his Comment filed
before this Court. Further, respondent did not
Issue: W/N he is entitled to separation pay? appeal the Labor Arbiter's reinstatement order.

Ruling: Contrary to the Court of Appeals' ruling, there is


nothing in the records showing any strained
Article 279 of the Labor Code of the Philippines relations between the parties to warrant the award
mandates the reinstatement of an illegally dismissed of separation pay. There is neither allegation nor
employee, to wit: proof that such animosity existed between
petitioner and respondent. In fact, petitioner
Security of Tenure. - x x x An employee who is
complied with the Labor Arbiter's reinstatement
unjustly dismissed from work shall be entitled to
order.
reinstatement without loss of seniority rights and
other privileges and to his full back wages, inclusive Considering that (1) petitioner reinstated
of allowances, and to his other benefits or their respondent in compliance with the Labor Arbiter's
monetary equivalent computed from the time his decision, and (2) there is no ground, particularly
compensation was withheld from him up to the time strained relations between the parties, to justify the
of his actual reinstatement. grant of separation pay, the Court of Appeals erred
in ordering the payment thereof, in lieu of
Thus, reinstatement is the general rule, while the
reinstatement.
award of separation pay is the exception.

In this case, petitioner claims that it complied with


the reinstatement order of the Labor 70. WENPHIL CORPORATION, vs.ALMER R. ABING
Arbiter.1wphi1 On 23 January 2008, petitioner sent and ANABELLE M. TUAZON-GARRY
respondent a notice informing him of the Labor
Arbiters decision to reinstate him. Accordingly, in Facts: Labor Arbiter Geobel A. Bartolabac ruled that
February 2008, respondent was assigned by the respondents had been illegally dismissed by
petitioner to Canlubang Sugar Estate, Inc. in Wenphil. According to him, the allegation of serious
Canlubang, Laguna, and to various posts thereafter. misconduct against the respondents had no factual
At the time of the filing of the petition, respondent and legal basis. Consequently, he ordered Wenphil
was assigned by petitioner to MD Distripark Manila, to immediately reinstate the respondents to their
Inc. in Bian, Laguna. respective positions or to equivalent ones. Also, he
ordered Wenphil to pay the respondents their
Respondent admits receiving a reinstatement notice backwages from February 3, 2000 until the date of
from petitioner. Thereafter, respondent was their actual reinstatement
assigned to one of petitioner's clients. However,
Issue: Is the CA correct?

Because of the unfavorable LA decision, Wenphil Ruling: Yes. As we held in the case of Garcia v.
appealed to the NLRC. In the meantime, the Philippine Airlines:
respondents moved for the immediate execution of
the LAs decision. The Court reaffirms the prevailing principle that even
if the order of reinstatement of the Labor Arbiter is
On October 29, 2001, Wenphil and the respondents reversed on appeal, it is obligatory on the part of the
entered into a compromise agreement before LA employer to reinstate and pay the wages of the
Bartolabac. They agreed to the respondents payroll dismissed employee during the period of appeal
reinstatement while Wenphils appeal with the NLRC until reversal by the higher court. It settles the view
was ongoing. Wenphil also agreed to pay the that the Labor Arbiter's order of reinstatement is
accumulated salaries of the respondents for the immediately executory and the employer has to
payroll. either re-admit them to work under the same terms
and conditions prevailing prior to their dismissal, or
The NLRC issued a resolution affirming LA to reinstate them in the payroll, and that failing to
Bartolabacs decision with modifications. Instead of exercise the options in the alternative, employer
ordering the respondents reinstatement, the NLRC must pay the employees salaries.
directed Wenphil to pay the respondents their
respective separation pay. Also, the NLRC found that
while the respondents had been illegally dismissed,
they had not been illegally suspended. Thus, the 71. Manila Water Company vs Del Rosario , G.R. No.
period from February 3 to February 28, 2000 during 188747, January 29, 2014- SAL
which the respondents were on preventive
suspension was excluded by the NLRC in the Facts: Del Rosario was employed as Instrument
computation of the respondents backwages. Technician by Metropolitan Waterworks and
Sewerage System (MWSS). Sometime in 1996,
CA reversed the NLRCs finding that the respondents MWSS was reorganized pursuant to Republic Act No.
had been illegally dismissed. According to the CA, 8041 or the National Water Crisis Act of 1995, and its
there was enough evidence to show that the implementing guidelines Executive Order No. 286.
respondents had been guilty of serious misconduct; Because of the reorganization, Manila Water
thus, their dismissal was for a valid cause. Also, the absorbed some employees of MWSS including Del
CA ruled that the NLRC committed grave abuse of Rosario. In August 1997, Del Rosario officially
discretion when it affirmed the LAs computed became an employee of Manila Water.
period which was from February 15, 2002 to
November 8, 2002. In arriving at this conclusion, the Sometime in May 2000, Manila Water discovered
CA cited the case of Pfizer v. Velasco where this that 24 water meters were missing in its stockroom.
Court ruled that even if the order of reinstatement Upon initial investigation, it appeared that Del
of the Labor Arbiter is reversed on appeal, it is Rosario and his co-employee were involved in the
obligatory on the part of the employer to reinstate pilferage and the sale of water meters to the
and pay the dismissed employees wages during the companys contractor. Consequently, Manila Water
period of appeal until reversal by the higher court. issued a Memorandum, directing Del Rosario to
The CA construed this "higher court" to be the CA, explain in writing within 72 hours why he should not
not the SC. be dealt with administratively for the loss of the said
water meters. In his letter-explanation, Del Rosario
The CA reasoned out that it was a "higher court" confessed his involvement in the act charged and
than the NLRC when it reversed the NLRCs rulings; pleaded for forgiveness, promising not to commit
thus, the period for computation should end when it similar acts in the future.
promulgated its decision reversing that of the NLRC,
and not on the date when the SC affirmed its In June 2000, Manila Water conducted a hearing to
decision afford Del Rosario the opportunity to personally
defend himself and to explain and clarify his
defenses to the charge against him. During the
formal investigation Del Rosario was found
responsible for the loss of the water meters and individual or collective agreement with the employer
therefore liable for violating Section 11.1 of the or voluntary employer policy or practice.
Companys Code of Conduct. Manila Water
proceeded to dismiss Del Rosario from employment. In exceptional cases, however, the Court has granted
separation pay to a legally dismissed employee as an
This prompted Del Rosario to file an action for illegal act of "social justice" or on "equitable grounds." In
dismissal claiming that his severance from both instances, it is required that the dismissal (1)
employment is without just cause. In his Position was not for serious misconduct; and (2) did not
Paper submitted before the labor officer, Del Rosario reflect on the moral character of the employee.
averred that his admission to the misconduct
charged was not voluntary but was coerced by the The attendant circumstances in the present case
company. Such admission therefore, made without considered, we are constrained to deny Del Rosario
the assistance of a counsel, could not be made basis separation pay since the admitted cause of his
in terminating his employment. dismissal amounts to serious misconduct. He is not
only responsible for the loss of the water meters in
Refuting the allegations of Del Rosario, Manila Water flagrant violation of the companys policy but his act
pointed out that he was indeed involved in the is in utter disregard of his partnership with his
taking of the water meters from the companys stock employer in the pursuit of mutual benefits.
room and of selling these to a private contractor for
personal gain. Invoking Section 11.1 of the That Del Rosario rendered 21 years of service to the
Companys Code of Conduct, Manila Water averred company will not save the day for him. Indubitably,
that such act of stealing the companys property is the appellate court erred in awarding separation pay
punishable by dismissal. The company invited the to Del Rosario without taking into consideration that
attention of this Court to the fact that Del Rosario the transgression he committed constitutes a serious
himself confessed his involvement to the loss of the offense. The grant of separation pay to a dismissed
water meters not only in his letter-explanation, but employee is determined by the cause of the
also during the formal investigation, and in both dismissal. The years of service may determine how
instances, pleaded for his employers forgiveness. much separation pay may be awarded. It is,
however, not the reason why such pay should be
The Labor Arbiter issued a Decision dismissing for granted at all.
lack of merit the complaint filed by Del Rosario who
was, however, awarded separation pay. According to In sum, we hold that the award of separation pay or
the Labor Arbiter, Del Rosarios length of service for any other kind of financial assistance to Del Rosario,
21 years, without previous derogatory record, under the nomenclature of compassionate justice, is
warrants the award of separation pay. not warranted in the instant case. A contrary rule
would have the effect of rewarding rather than
Issue: WON the court erred in awarding separation punishing an erring employee, disturbing the noble
pay to respondent concept of social justice.

Ruling: YES. As a general rule, an employee who has


been dismissed for any of the just causes
enumerated under Article 282 of the Labor Code is 72. G.R. No. 167286
not entitled to a separation pay. Section 7, Rule I,
Book VI of the Omnibus Rules implementing the February 5, 2014INTERNATIONAL SCHOOL MANILA
Labor Code provides: AND/OR BRIAN McCAULEY, Petitioners,
vs.
Sec. 7. Termination of employment by employer. INTERNATIONAL SCHOOL ALLIANCE OF EDUCATORS
The just causes for terminating the services of an (ISAE) AND MEMBERS REPRESENTED BY RAQUEL
employee shall be those provided in Article 282 of DAVID CHING, PRESIDENT, EVANGELINE SANTOS,
the Code. The separation from work of an employee JOSELYN RUCIO AND METHELYN FILLER,
for a just cause does not entitle him to the Respondents- CEL
termination pay provided in the Code, without
prejudice, however, to whatever rights, benefits and Facts: Respondent Evangeline Santos was a faculty of
privileges he may have under the applicable the International School Manila since 1978. In 1992
she applied for a leave of absence. When she In the collective bargaining agreement (CBA)
returned in 1993, only one teaching load for Spanish between the School and ISAE for the years 1992-
was available, thus she agreed to teach 4 Filipino 1995, Section 13 of Appendix A thereof expressly
classes. As per evaluation during the year 1993, states that "termination of employment shall be in
respondents evaluation stated that she needed accordance with the laws of the Philippines as
improvement in key areas such as use of effective presented in the LABOR CODE.
questioning techniques and enforcement of
academic and classroom behavior among others. In Century Iron Works, Inc. v. Baas, the concept of
The following school year, Santos expressed that she gross and habitual neglect of duties is explained as:
will be teaching for the school year and that she did
not prefer a change in the teaching assignments. For Gross negligence connotes want or absence of or
that school year, she was again evaluated and her failure to exercise slight care or diligence, or the
evaluation results mirrored those areas in which she entire absence of care. It evinces a thoughtless
needed improvement. The situation was the same disregard of consequences without exerting any
for school year 1995-1996. In 1996, a Professional effort to avoid them. Fraud and willful neglect of
Growth Plan was signed by Santos wherein she duties imply bad faith of the employee in failing to
undertook to focus and improve on the specifically perform his job, to the detriment of the employer
stated areas of her teaching that she need to and the latters business. Habitual neglect, on the
improve on. Phase 1 was Planning. But even with the other hand, implies repeated failure to perform
Plan and the series of meetings and consultations ones duties for a period of time, depending upon
conducted her benefit, her over-all performance did the circumstances.
not improve. In fact, 8 months into the
implementation of the Professional Growth Plan, she
was still in Phase 1. In April 10, 1997, the school then In Lim v. National Labor Relations Commission:
wrote a letter to Santos asking her to explain why
her services should not be terminated in view of her Gross inefficiency falls within the purview of "other
performance way below the standards set by said causes analogous to the foregoing," and constitutes,
school. Santos was given the chance to answer. A therefore, just cause to terminate an employee
meeting was then conducted where Santos was under Article 282 of the Labor Code. One is
allowed to bring counsel or representative. Santos analogous to another if it is susceptible of
was accompanied by Raquel Ching, President of the comparison with the latter either in general or in
International School Alliance of Educators. Santos some specific detail; or has a close relationship with
was being charged by the school with gross the latter. "Gross inefficiency" is closely related to
inefficiency or negligence in the performance of "gross neglect," for both involve specific acts of
duties. An administrative investigation followed. The omission on the part of the employee resulting in
committee who conducted the investigation damage to the employer or to his business.
recommended that the employment of Santos
cannot be continued. Santos was informed in a letter In Buiser vs. Leogardo, the Court ruled that failure to
of her termination effective June 7, 1997, based on observe prescribed standards of work, or to fulfill
the finding of the committee that despite 3 years of reasonable work assignments due to inefficiency
consultations with her supervisors, no appreciable may constitute just cause for dismissal.
improvement was seen in her performance.
Viewed in light of the above doctrines, the Court is
Issue: Was Santos illegally dismissed? not convinced that the actuations of Santos
complained of by the petitioners constituted gross
Ruling: No. To constitute a valid dismissal from and habitual neglect of her duties.
employment, two requisites must concur: (1) the
dismissal must be for any of the causes provided in What can be gathered from a thorough review of the
Article 282 of the Labor Code; and, (2) the employee records of this case is that the inadequacies of
must be given an opportunity to be heard and to Santos as a teacher did not stem from a reckless
defend himself. disregard of the welfare of her students or of the
issues raised by the School regarding her teaching.
Far from being tainted with bad faith, Santoss
failings appeared to have resulted from her lack of (ii) A hearing or conference during which the
necessary skills, in-depth knowledge, and expertise employee concerned, with the assistance of counsel
to teach the Filipino language at the standards if he so desires is given opportunity to respond to
required of her by the School. the charge, present his evidence, or rebut the
evidence presented against him.
The petitioners had sufficiently proved the charge of
gross inefficiency, which warranted the dismissal of (iii) A written notice of termination served on the
Santos from the School. employee, indicating that upon due consideration of
all the circumstances, grounds have been
The Court enunciated in Pea v. National Labor established to justify his termination
Relations Commission73 that "it is the prerogative of
the school to set high standards of efficiency for its
teachers since quality education is a mandate of the
Constitution. As long as the standards fixed are The School complied with the above requirements.
reasonable and not arbitrary, courts are not at After a thorough evaluation of Santoss
liberty to set them aside." Moreover, the prerogative performance, the School held a series of conferences
of a school to provide standards for its teachers and and meetings with Santos, in order to improve her
to determine whether these standards have been performance. On March 29, 1996, the School
met is in accordance with academic freedom, which required Santos to undertake a Professional Growth
gives the educational institution the right to choose Plan. Thereafter, when the intervention of the
who should teach. School failed to yield any considerable improvement
on Santos, McCauley wrote her a letter on April 10,
The Court finds that, not only did the petitioners 1997, which required her to explain in writing within
documentary evidence sufficiently prove Santoss forty-eight (48) hours why her employment should
inefficient performance of duties, but the same also not be terminated in view of her failure to meet the
remained unrebutted by respondents own standards of the School on very specific areas of
evidence. On the contrary, Santos admits in her concern. On April 16, 1997, Santos responded to
pleadings that her performance as a teacher of McCauleys letter, asking why she was being
Filipino had not been satisfactory but she prays for required to explain. On April 21, 1997, McCauley
leniency on account of her prior good record as a wrote Santos a letter informing her that an
Spanish teacher at the School. Indeed, even the administrative investigation would be conducted on
Labor Arbiter, the NLRC and the Court of Appeals April 23, 1997 where she would be given the
agreed that Santos was not without fault but the opportunity to be heard. On April 23, 1997, an
lower tribunals deemed that termination was too administrative investigation was conducted. Santos
harsh a penalty. appeared therein with the assistance of ISAE
President Ching. In a letter dated May 29, 1997, the
Nonetheless, the Court finds that petitioners had School informed Santos of its decision to terminate
satisfactorily discharged the burden of proving the her employment on the ground of her failure to
existence of gross inefficiency on the part of Santos, meet the standards of the School, which as
warranting her separation from the school. discussed was tantamount to gross inefficiency.
As regards the requirements of procedural due
process, Section 2(d) of Rule 1 of The Implementing
Rules of Book VI states that:

For termination of employment based on just causes


as defined in Article 282 of the Labor Code:

(i) A written notice served on the employee


specifying the ground or grounds for termination,
and giving said employee reasonable opportunity
within which to explain his side.

Potrebbero piacerti anche